image

1. Энтропия измеряет не беспорядок, а вероятность


Идея о том, что энтропия – это мера беспорядка, совсем не помогает разобраться в вопросе. Допустим, я делаю тесто, для чего я разбиваю яйцо и выливаю его на муку. Затем добавляю сахар, масло, и смешиваю их до тех пор, пока тесто не становится однородным. Какое состояние является более упорядоченным – разбитое яйцо и масло на муке, или получившееся тесто?

Я бы сказала, что тесто. Но это состояние с большей энтропией. А если вы выберете вариант с яйцом на муке – как насчёт воды и масла? Энтропия выше, когда они разделены, или после того, как вы их яростно потрясёте, чтобы смешать? В данном примере энтропия выше у варианта с разделёнными веществами.

Энтропия определяется как количество “микросостояний”, дающих одно и то же “макросостояние”. В микросостояниях содержатся все детали по поводу отдельных составляющих системы. Макросостояние же характеризуется только общей информацией, вроде “разделено на два слоя” или “в среднем однородное”. У ингредиентов теста есть много разных состояний, и все они при смешивании превратятся в тесто, однако очень мало состояний сможет при смешивании разделиться на яйца и муку. Поэтому, у теста энтропия выше. То же работает для примера с водой и маслом. Их легче разделить, тяжелее смешать, поэтому у разделённого варианта энтропия выше.

2. Квантовая механика применима не только к малым расстояниям, её просто тяжелее наблюдать на больших дистанциях


В теории квантовой механики нет никаких ограничений, согласно которым она работала бы только на коротких дистанциях. Просто так получается, что наблюдаемые нами крупные объекты состоят из множества более мелких, чьё тепловое движение уничтожает все типичные квантовые эффекты. Этот процесс называется декогеренцией, и именно из-за него мы обычно не видим проявлений квантовой механики в повседневной жизни.

Но квантовые эффекты измеряли в экспериментах, протянувшихся на сотни километров, и они могут работать и на больших расстояниях в достаточно стабильной и холодной среде. Они могут распространяться даже на всю галактику целиком.

3. Тяжёлые частицы распадаются не до состояния с минимальной энергией, а до состояния с максимальной энтропией


Энергия сохраняется. Поэтому идея о том, что любая система пытается минимизировать энергию, не имеет смысла. Причина, по которой тяжёлые частицы распадаются, когда могут, состоит в том, что они это могут. Если у нас есть одна тяжёлая частица (допустим, мюон), она может распасться на электрон, мюонное нейтрино и электронное антинейтрино. Возможен и противоположный процесс, но для него требуется, чтобы в одном месте собрались три продукта распада. Следовательно, вероятность его мала.

Но это не всегда так. Если поместить тяжёлые частицы в достаточно горячий “суп”, то синтез и распад могут достичь равновесия, при котором будет существовать ненулевое количество тяжёлых частиц.

4. Линии в диаграммах Фейнмана не изображают пути движения частиц, это просто вспомогательные рисунки для сложных вычислений


Периодически я получаю электронные письма от людей, замечающих, что во многих диаграммах Фейнмана линиям назначаются импульсы. А поскольку все знают, что нельзя в одно и то же время измерить местоположение и импульс частицы с произвольной точностью, в линиях движения частиц нет смысла. Отсюда вроде бы следует, что физика частиц неверна!

Но с физикой частиц всё в порядке. Диаграммы Фейнмана бывают разные, и те, которые обозначают импульсы, предназначены для импульсного пространства. В данном случае эти линии никак не связаны с путями движения частиц. Вообще. Это просто способ изображать некоторые виды интегралов.

В некоторых диаграммах Фейнмана линии на самом деле изображают возможные пути, по которым может пойти частица, но и в этом случае диаграмма не говорит о том, что на самом деле сделает частица. Для этого приходится проводить вычисления.

5. Квантовая механика нелокальна, но её нельзя использовать для нелокального переноса информации


Квантовая механика порождает нелокальные связи, количественно более сильные, чем связи в неквантовых теориях. Это Эйнштейн и называл “пугающим дальнодействием“.

Увы, квантовая механика также по сути своей случайна. Поэтому, хотя у нас и есть эти удивительные нелокальные связи, их нельзя использовать для передачи сообщений. Квантовая механика на самом деле полностью совместима с ограничением скорости света по Эйнштейну.

6. Квантовая гравитация начинает играть роль в ситуациях с высокой кривизной, а не с короткими расстояниями


Если оценивать силу эффектов квантовой гравитации, можно обнаружить, что они перестают быть пренебрежимо малыми в случае, когда кривизна пространства-времени сравнима с величиной, обратной квадрату планковской длины. Это не означает, что эти эффекты можно будет увидеть на дистанциях, близких к планковской длине. Мне кажется, что путаница возникает из-за термина “планковская длина”. Планковская длина – это единица длины, а не длина чего-то конкретного.

Важно здесь то, что утверждение “приближение кривизны к обратному квадрату планковской длины” не зависит от наблюдателя. Она не зависит от скорости вашего движения. Проблема идеи о том, что квантовая гравитация начинает играть роль на коротких расстояниях, состоит в том, что она несовместима со Специальной теорией относительности.

В СТО длины могут сокращаться. Для достаточно быстро движущегося наблюдателя Земля будет выглядеть как блин с шириной меньше, чем планковская длина. И это будет означать, что мы либо должны заметить эффекты квантовой гравитации, либо СТО неверна. Свидетельства говорят против обоих предположений.

7. Атомы не расширяются вместе с расширением Вселенной. Как и Москва


Расширение Вселенной идёт невероятно медленно и оказывает очень слабое воздействие. На системы, связанные воедино при помощи взаимодействий, превышающих по силе расширение, оно не влияет. Системы, которые расширение способно разорвать на части, превышают по размеру скопления галактик. Сами же скопления удерживаются вместе благодаря гравитации. Как и галактики, солнечные системы, планеты и, естественно, атомы. Последние удерживаются вместе благодаря атомным взаимодействиям, куда как более сильным, чем расширение Вселенной.

8. Червоточины – это научная фантастика, а чёрные дыры – нет


Свидетельства, полученные из наблюдений за чёрными дырами, чрезвычайно убедительны. Астрофизики подтверждают наличие чёрных дыр множеством способов.

Простейший способ – подсчитать, сколько массы необходимо собрать в определённом объёме пространства, чтобы в результате получить такое движение близлежащих к нему объектов, которое наблюдается в реальности. Это само по себе, конечно, не говорит о том, есть ли у тёмного объекта, влияющего на видимые объекты, горизонт событий. Однако можно увидеть разницу между горизонтом событий и твёрдой поверхностью, исследуя излучения, испускаемое тёмным объектом. Также чёрные дыры можно использовать в качестве чрезвычайно мощных гравитационных линз, чтобы проверить их соответствие предсказаниям Общей теории относительности Эйнштейна. Поэтому физики с огромным интересом ждут данных от Телескопа горизонта событий [проект, объединяющий множество радиотелескопов во всего мира для изучения центральной чёрной дыры Млечного пути / прим. перев.].

Возможно, самое важное, что нам известно – чёрные дыры являются типичным конечным состоянием коллапса звёзд определённых типов. В ОТО их легко получить и тяжело избежать.

С другой стороны, червоточины – это деформации пространства-времени, о возникновении которых в результате естественных процессов нам ничего не известно. Также для их наличия требуется отрицательная энергия, которую никто никогда не видел, и по поводу существования которой у многих физиков есть большие сомнения.

9. В чёрную дыру можно упасть за конечное время. Это просто будет выглядеть так, будто занимает вечность


При приближении к горизонту событий время замедляется, но это не значит, что вы заканчиваете падение до того, как достигнете горизонта событий. Это замедление увидит только наблюдатель, расположенный на некотором расстоянии. Можно подсчитать, сколько времени займёт падение в чёрную дыру по часам падающего. Результат получается конечным. В чёрную дыру упасть можно. Просто ваш друг снаружи этого никогда не увидит.

10. Во Вселенной в целом энергия не сохраняется, но этот эффект настолько мал, что его не получится засечь


Я говорила, что энергия сохраняется – но это утверждение верно только в определённом приближении. Оно было бы полностью верным во вселенной, в которой пространство не менялось бы со временем. Но мы знаем, что в нашей Вселенной пространство расширяется, и это расширение нарушает закон сохранения энергии.

Однако это нарушение настолько крохотное, что его не заметить ни в каком эксперименте, проводимом на Земле. Чтобы его заметить, нужно наблюдать очень долго за очень большими расстояниями. Если бы этот эффект был сильнее, мы бы уже давно заметили, что Вселенная расширяется! Поэтому не вините Вселенную в ваших счетах за электричество, а просто закрывайте окно, когда включаете кондиционер.

Больше статей на научно-популярную тему (19 за последний месяц) вы можете найти на сайте Golovanov.net. Подписывайтесь на обновления по e-mail, через RSS или канал Яндекс.Дзен.

По многочисленным просьбам реализована возможность поддержать проект материально.

Комментарии (223)


  1. netricks
    05.10.2018 12:08

    За объяснение энтропии спасибо.


    1. 5oclock
      05.10.2018 13:43
      +1

      По мне так как-раз с энтропией объяснение какое-то невнятное.
      Про масло и воду я вообще не понял.


      1. BubaVV
        05.10.2018 14:28

        Дж. Ффорде. Беги, Четверг, беги, или Жесткий переплет
        Он достал с одной из многочисленных полок банку из-под варенья и вручил ее мне. Содержимое ее с виду составляли рис и чечевица — примерно пополам.

        — Спасибо, я не голодна.

        — Нет-нет. Я называю это устройство энтроскопом. Встряхни-ка его.

        Я встряхнула банку, и рис с чечевицей перемешались в случайном порядке, как обычно и бывает.

        — Ну и что? — спросила я.

        — Ничего необычного, — ответил Майкрофт. — Стандартное распределение, уровень энтропии нормален. Встряхивай ее почаще. Если произойдет падение энтропии, ты это увидишь: рис и чечевица распределятся не так хаотично, а значит, стоит ожидать совершенно невероятных совпадений.


        1. agat000
          05.10.2018 14:45

          Я так делал. С шариковыми конфетами ещё в детстве. Никогда они равномерно не перемешиваются. Всегда получаются сгустки.


          1. xenon
            06.10.2018 02:47

            Так сгустки и нормально. Было бы очень странно, если бы из 6 бросков кубика, стабильно выпал бы каждый из 6 вариантов по 1 разу (это бы означало, что 6 бросок не случаен). Так же, как и 500 орлов и 500 решек в тысяче бросков монетки.


            1. Igor_O
              06.10.2018 19:12

              Было бы очень странно, если бы из 6 бросков кубика, стабильно выпал бы каждый из 6 вариантов по 1 разу

              А что в этом такого? Да, вероятность не очень большая, если я не обсчитался, то что-то типа 1 к 22 квадриллионам (при условии, что выпадение единицы на шестом броске мы игнорируем и начинаем бросать снова с 1. Понятно, что если на n-ном броске выпала единица и мы считаем это началом серии, то там уже будет 1 из примерно 4 квадриллионов, опять же, если я не обсчитался, а я сейчас болею и перепроверять расчеты нет сил).
              В том и суть теории вероятности. Какой бы невероятной не казалась бы последовательность, есть вероятность, что выпадет именно она.
              Исправление/дополнение: я что-то не обратил внимания, что вы вообще простой вариант выбрали, просто каждый из по одному разу, а не последовательно 1-2-3-4-5-6. Если просто каждый по разу, то там вообще очень высокая вероятность. Что-то типа полутора процентов!

              500 орлов и 500 решек в тысяче бросков

              Вероятность, что их будет ровно по 500 — достаточно высока. Все же, у нас вероятность выпадения орла и решки по 1/2, чем длиннее серия, тем ближе будут результаты орла и решки.
              А как вам вероятность угадывания результата примерно 100 бросков монетки без единой ошибки? А я такое однажды видел своими глазами…
              Более того, лет тому с десяток назад читал в какой-то статье про информационную безопасность, что в честных «физических» генераторах случайных чисел, делали системы для отбрасывания недостаточно случайных последовательностей. Т.к. из-за честности генератора, есть вероятность получить ключ шифрования из только нулей или только единиц.


              1. Victor_koly
                06.10.2018 20:22

                Вероятность выпадения 500 «решек» в бесконечности серий по 1000 бросков описывается законом больших чисел.
                Принимаем x = 1 <-> «решка», x = 0 <-> «орел».
                Вероятность каждого события 1/2, среднее 1/2, дисперсия sigma^2 = ((1/2)^2+(1/2)^2)/2 = 0.25.
                Значит «среднее число решек» в множестве экспериментов будет описываться приблизительно распределением таким, что это будет распределение Гаусса с
                = 250 и какой-то там сигма (нужно честно посчитать сумму 500 слагаемых будет?).


          1. BigBeaver
            06.10.2018 09:51

            Это потому, что случайно и равномерно — не синонимы. У строго равномерного распределения будет более высокая энтропия.


            1. BigBeaver
              06.10.2018 19:11

              В смысле, более низкая. Надо меньше работать и больше спать)


        1. JC_IIB
          05.10.2018 15:28
          +2

          Ффорде великолепен. Больше всего у него мне нравится «Полный вперед назад, или оттенки серого», жаль, что он так и не продолжил эту книгу. Сама концепция мира, в котором твое положение зависит от способности воспринимать цвета — это что-то с чем-то.


        1. agarus
          05.10.2018 16:11

          Розенкранц и Гильденстерн обнаружили ее нарушенияе с помощью монетки :)


      1. JTG
        05.10.2018 15:00

        Я тоже несколько запутан. Видимо, имеется в виду, что взвесь масла в воде без внешнего воздействия отстоится и разделится на компоненты. Согласно закону неубывания энтропии выходит, что раз система сама пришла из взвеси в разделённое состояние, то последнее имеет большую энтропию. Это довольно контринтуитивно, если мы считаем энтропию мерой неупорядоченности, ведь «вперемешку» кажется более хаотичным состоянием.


        1. BubaVV
          05.10.2018 15:33
          +1

          Тут вступает в дело свободная энергия границы раздела фаз — при расслоении она заметно уменьшается, и это перебивает энтропийный эффект


        1. 5oclock
          05.10.2018 17:41

          То что мы обычно понимаем под словом "отстоится" — подразумевает воздействие силы тяжести.


          1. Alternator
            05.10.2018 23:17
            +1

            Полагаю для разделения масла и воды их плотности и сила гравитации играют роль только в вопросе «кто всплывет, кто потонет».
            А сам механизм разделения обоснован исключительно силой поверхностного натяжения, и несмачиваемостью.
            Если налить в блюдце воды, и капнуть пару капель масла на поверхность, то при встрече они слипнутся, и сила тяжести тут не при чем.
            Так что и в отсутствие силы гравитации масло с водой разделятся


            1. arheops
              06.10.2018 01:28

              В отсутсвие силы гравитации на это уйдет очень уж много времени. Если капля воды достаточно большая — типа Луны, то может вообще никогда(капли масла внутри могут летать туда сюда под действием приливных сил).


              1. old_gamer
                06.10.2018 02:34

                Если капля воды будет размером с Луну, то тут уж никак не сказать

                В отсутсвие силы гравитации


            1. andyudol
              06.10.2018 07:10

              Если налить в блюдце воды, и капнуть пару капель масла на поверхность, то при встрече они слипнутся, и сила тяжести тут не при чем.


              Жир в супе ведёт себя по другому. Если его немного, то сам он в один слой не сливается. Надо разрушать границы между каплями ложкой. Это развлечение я в детстве очень любил.


              1. maxzhurkin
                07.10.2018 15:39
                +1

                Так вот откуда глобалисты берутся!


                1. tvr
                  07.10.2018 15:47

                  Из детства?


                  1. BigBeaver
                    07.10.2018 16:05

                    Из супа.


      1. netricks
        05.10.2018 15:25
        +1

        А по мне очень доходчиво…

        Фактически это о том, что у системы есть некий наиболее вероятный класс состояний, к которому она стремиться при бесконечной последовательности случайных изменений (можно считать, что действует закон больших чисел). Для риса и чечевицы — это мешанина. Для воды и масла — это разделение на две фракции.

        То есть, по сути, энтропия — это не о физическом процессе вообще, а о круговороте возможных состояний системы. И теперь мне наконец-то понятна энтропия в информатике.


        1. BigBeaver
          05.10.2018 22:01

          Автор просто сам «плавает» и разводит полемику на пустом месте. Почему пример с маслом не корректен, уже написали.


      1. sena
        05.10.2018 18:07

        Всё то что необратимо или тяжело обратимо (или маловероятно) повышает энтропию. Сжечь бензин легко, синтезировать бензин из воды и углекислоты — тяжело. Состариться — легко, помолодеть — тяжело.


        1. zomby
          06.10.2018 08:25

          Не получается у меня ухватить суть концепции. Существуют необратимые изменения, это я понимаю. Если слить воду из двух стаканов в один, невозможно будет сказать где сколько изначально было. Следовательно, имея состояние системы на некоторый момент, невозможно построить модель прошлого состояния. А модель будущего можно, если мир детерминирован (истинно случайных событий не происходит). Следовательно будущее содержит меньше информации. Но ведь случайные события происходят (распад ядер, например).


          1. sena
            07.10.2018 12:48

            Если слить воду из двух стаканов в один, невозможно будет сказать где сколько изначально было. Следовательно, имея состояние системы на некоторый момент, невозможно построить модель прошлого состояния. А модель будущего можно, если мир детерминирован (истинно случайных событий не происходит).
            Если бы мир был полностью детерминирован, то обратив движение каждой молекулы и фотона, можно было бы рассчитать предыдущее состояние, вплоть до двух стаканов, почему нет?

            истинно случайных событий не происходит

            Что есть «истинно случайное событие»? Событие, которое мы не можем предсказать? Но ведь всегда можно сказать, что мы просто не знаем, как это рассчитать, по какой причине это событие происходит, но это не значит что этой причины нет.
            Следовательно будущее содержит меньше информации.

            Я думаю что и это неверное утверждение. Количество информации как раз сохраняется, как и энергия. Изменяется её упорядоченность.


    1. gasizdat
      05.10.2018 14:47

      Вроде в институте меня учили, что энтропия системы, находящейся в состоянии А определяется числом путей, по которым система может перейти в это состояние из других состояний (B, C, D etc). Это если очень грубо. Поэтому, чем больше способов перейти в какое-то состояние, тем выше его энтропия и наоборот. Как правило поэтому переход яйца в тесто — это переход из состояния с низкой энтропией в состояние с высокой.


      1. netricks
        05.10.2018 15:29

        Мне кажется, пример с тестом притянут за уши. Но вода и масло, и чечевица с рисом из комментария выше — таки хорошие примеры.


        1. 4tlen
          05.10.2018 21:48

          Еще лучше пример: упорядоченная колода карт, подбрасываем, карты хаотично падают — энтропия возросла. Потому что упасть упорядоченно 1 шанс (ну потому что упорядоченное состояние колоды оно одно), упасть беспорядочно миллиарды шансов. Поэтому беспорядок или энтропия как его мера всего навсего более вероятен.


          1. DrSmile
            06.10.2018 06:37

            Пример с картами не удачен, потому что когда карты уже упали и мы их видим, то энтропия нулевая, независимо от того, как конкретно они лежат. Энтропия относится только к вероятностным распределениям, это мера нашего незнания о системе. Поэтому энтропия максимальная, когда мы бросили карты, но на результат не смотрели; а если посмотрим, то обнулим энтропию. Т. е. когда карты лежат совершенно случайно (и мы ничего не знаем), то энтропия максимальна, а когда лежат конкретным образом — то нулевая.


          1. BigBeaver
            06.10.2018 09:56

            Вы удивитесь, но вероятность падения любым строго заданным наперед образом будет примерно сопоставима, если не равна.


            1. 4tlen
              06.10.2018 10:24

              За порядок принимается начальное состояние, колода. И именно в таком же состоянии упасть шанс 1 к 1…000. Не настаиваю что это самое лучшее объяснение.


              1. BigBeaver
                06.10.2018 10:44

                По-моему, здесь просто вопросы к определению порядка.


              1. juray
                06.10.2018 16:14

                Если мы начинаем разбирать по конкретным картам (как выразился BigBeaver — «строго заданным наперед образом»), то это уже микросостояние, распределение вероятностей которых если и не равномерное, то стремится к таковому. То есть вероятность группировки в колоду такая же, как и «король треф упал в точку (x1,y1), дама пик в (x2,y2), и т.д.».
                А вот вероятность макросостояний распределяется неравномерно из-за неравномерной группировки микросостоянии. Макросостояние «упасть колодой» образуется куда меньшим числом микросостояний, чем состояние «упасть россыпью».


                1. 4tlen
                  07.10.2018 14:10

                  Макросостояние «упасть колодой» образуется куда меньшим числом микросостояний, чем состояние «упасть россыпью»
                  Собственно это объяснение через микросостояния, а не через вероятность. Но суть одна.


                  1. juray
                    07.10.2018 17:10

                    Ну так вероятность макросостояния зависит от числа микросостояний, которыми это макросостояние может быть образовано.


        1. BlessYourHeart
          06.10.2018 00:05

          Да уж. Яйца тут мешаются :)


          Я себе нашел пример для подумать: в норме нашей вселенной (надеюсь на эту норму) энергия переходит от более горячего тела к более холодному. Это минимальная энтропия процесса, так как вероятность иных исходов по дефолту минимальна. Если мы применили силу (например электричество) к этому процессу, то можем обратить его. Будет ли у обращённого процесса бОльшая энтропия? Вроде вариант в этой замкнутой системе с подачей электричества только один. Наверное энтропия зависит от системы координат.
          И яйца тут не причем.


    1. kalininmr
      05.10.2018 16:58

      можно несколько проще обьяснить:
      «всё падает вниз» — наивысшая энтропия — это наиболее равномерное состояние.


    1. Cast_iron
      06.10.2018 10:11

      Насколько я помню в РФ в 90-х про энтропию не рассказывали в школе. В учебнике по физике 10 класса Мякишева 2010 года есть глава о вероятности состояний, но понятие энтропии там не рассматривается.
      Сам я узнал о ней из учебника для ВУЗов.


  1. Lazytech
    05.10.2018 13:16

    Что касается пункта 8, среди комментариев к оригинальной версии статьи нашелся и такой:

    Daniel Wagner said…

    In your point 8, " Wormholes are science fiction, black holes are not", you said the wormholes need negative mass. Perhaps you missed this: arxiv.org/abs/1807.04726

    Аннотация к статье на arXiv.org (на английском)
    Traversable wormholes in four dimensions
    Juan Maldacena, Alexey Milekhin, Fedor Popov
    (Submitted on 12 Jul 2018 (v1), last revised 29 Aug 2018 (this version, v2))

    We present a wormhole solution in four dimensions. It is a solution of an Einstein Maxwell theory plus charged massless fermions. The fermions give rise to a negative Casimir-like energy, which makes the wormhole possible. It is a long wormhole that does not lead to causality violations in the ambient space. It can be viewed as a pair of entangled near extremal black holes with an interaction term generated by the exchange of fermion fields. The solution can be embedded in the Standard Model by making its overall size small compared to the electroweak scale.

    Вольный перевод названия статьи:
    Проходимые червоточины в четырехмерном пространстве


  1. andersong
    05.10.2018 13:57

    Преподаватель химии в ВУЗе утверждала, что понятие энтропии вне химии не имеет смысла. Что она имела ввиду?


    1. Zenitchik
      05.10.2018 14:52

      Скорее, вне термодинамики. Мы, например, с энтропией на физической химии сталкивались, когда считали тепловой эффект реакции и направление её течения.

      Я оттуда для себя вынес, что энтропия — это такая термодинамическая функция, которая используется в ряде расчётов. Ассоциировать её с чем-то наблюдаемым (хаосом, вероятностью и т.п.) — проблематично и вряд ли нужно.


    1. vanxant
      05.10.2018 14:56

      Что у нее совсем плохо с математикой и основами информатики.
      ЗЫ. На самом деле, не совсем так. Термодинамическая и информационная энтропии — разные сущности, хотя и связанные через принцип Ландауэра.


      1. DrSmile
        05.10.2018 16:00

        Термодинамическая и информационная энтропии — разные сущности
        Почему разные? Термодинамическая энтропия — это информационная, примененная к статистическому ансамблю. Условно, информационная энтропия — это мера нашего незнания относительно некоторого случайного распределения, а если это распределения из области статфизики, то получается энтропия термодинамическая.


        1. babayota_kun
          05.10.2018 18:55

          Отсюда, вроде, следует, что пример в статье неверен. Вероятность существования смеси воды и масла значительно меньше, чем вероятность существования воды и масла отдельно. Как следствие, у смеси энтропия выше.
          Равно как энтропия выше у теста, потому что вероятность существования ингредиентов в форме теста ниже, чем по отдельности.


          1. DrSmile
            06.10.2018 06:21

            В вероятность входит энергия (или другой термодинамический потенциал, в зависимости от граничных условий) в виде exp(?E/T), поэтому при достаточной разнице энергий (относительно температуры) этот энергетический вклад может перевесить вклад числа микросостояний, как получается в случае с маслом.


      1. Fracta1L
        06.10.2018 08:23

        Термодинамическая и информационная энтропии — разные сущности

        Нифига не разные, информация это так-то некоторым образом упорядоченная материя.


  1. DSolodukhin
    05.10.2018 14:09

    и это расширение нарушает закон сохранения энергии.

    Вот тут я бы поспорил с формулировкой. Согласно теореме Нётер законы сохранения являются следствием какого-либо вида симметрии, в частности, закон сохранения энергии следует из однородности времени. Проблема в том, что в ОТО время неоднородно, и закон сохранения энергии может быть выражен только локально. Но это проблема математическая, т.е. мы не можем формально выразить, что энергия сохраняется. Да мы вообще не можем сформулировать определение энергии, которое было бы применимо ко всей вселенной!
    На практике это означает, что мы вообще говоря, не имеем права говорить о том сохраняется ли энергия во всей вселенной или она куда-то утекает. По крайней мере, пока не появится новая теория гравитации, которая решит эту проблему.


    1. Zenitchik
      05.10.2018 14:53

      Проблема в том, что в ОТО время неоднородно, и закон сохранения энергии может быть выражен только локально.

      Однако, с точки зрения любого наблюдателя — энергия сохраняется.


      1. vanxant
        05.10.2018 15:04

        Не любого, а только находящегося в инерциальной системе отсчета.
        Подсказка: в нашей Вселенной абсолютно инерциальных СО нет.


        1. Zenitchik
          05.10.2018 15:28

          Не любого, а только находящегося в инерциальной системе отсчета.

          Разве? В НЕИСО просто есть инерционные силы. С их учётом — всё должно сходиться.


          1. vanxant
            05.10.2018 16:30

            Да банально же. Вы сидите в набирающем ход поезде (очевидно, неинерциальной СО). Земля за окошком всё набирает и набирает ход (т.е. энергию и импульс). Помножьте на массу Земли, вычтите мощность паровоза… Сходится?


            1. Zenitchik
              05.10.2018 16:34

              На первый взгляд — хочется согласиться. Но на второй — в неИСО есть сила инерции, которая разгоняет Землю. Раз есть сила, то должно быть и потенциальное поле, так?
              Правда, моей квалификации недостаточно, чтобы сформулировать, что такое потенциальная энергия инерционной силы.


            1. Taus
              05.10.2018 17:30

              Очевидный факт, что кинетическая энергия тела зависит от выбора системы отсчёта. И конечно всё сходится.


              1. vanxant
                05.10.2018 17:46
                +1

                Ну где сходится-то? Вот я выбрал систему отсчёта, связанную со мной, сидящим в ускоряющемся поезде. Я знаю массу Земли за окном, я вижу (по телеграфным столбам), что Земля ускоряется. Я знаю, что ускорение вызвано паровозом и я знаю мощность этого паровоза. Но — вот хоть убей не сходится.


                1. pulsatrix
                  05.10.2018 18:53

                  Не надо ни кого убивать, все сходится. Когда поезд затормозит, выделится то же количество энергии, что и при разгоне. А энергию земли, вычисленную вами, использовать нельзя — это виртуальная энергия ваших домыслов.


                  1. vanxant
                    05.10.2018 19:15

                    Если некто высунет свою глупую голову в окошко движущегося поезда, он очень быстро убедится в реальности «виртуальной энергии моих домыслов».


                    1. pulsatrix
                      05.10.2018 19:23

                      Ну да, на это потратится энергии не больше, чем есть у поезда. Грубо говоря, если поезд столкнётся со стеной, намертво вкопанной в землю, то энергии выделится не больше, чем масса поезда умноженная на скорость, относительно земли, в квадрате.


                      1. vanxant
                        05.10.2018 19:32

                        У поезда в моей СО нет никакой кинетической энергии. Согласно моей линейке и часам поезд стоит на месте, а едет вокзал.


                        1. pulsatrix
                          05.10.2018 20:04

                          Я как наблюдатель крушения поезда, с вами не соглашусь. У поезда энергия была, и много. И даже земля чуть быстрее вращаласть навстречу, но стена все вернула на свои места. В случае с поездом — инерциальная система отсчета — центр масс земли и поезда. А вы со своей линейкой и часами не там сидите.


                        1. Comod
                          05.10.2018 20:50
                          -1

                          Это все равно, что взять Васю и Петю, Вася пашет весь день, а Петя спит и получает зарплату еще и думает, что это спит оказывается Вася, это и есть вся абсурдность классической СТО. На самом деле само пространство является инерциальной системой отсчета, относительно которого все в природе и происходит, пространство всегда движется со скоростью света и это есть время.


                          1. Fracta1L
                            06.10.2018 08:30

                            Это что-то из физики для пролетариев?


                        1. BigBeaver
                          05.10.2018 22:09

                          Сохраняется полная энергия, а не кинетическая, вот и всего делов. ЕЕ разложение на сумму потенциальной и кинетической зависит от СО, так же, как от этого зависит разложение ЭМ поля на магнитное и электрическое. Если посчитать все правильно и аккуратно, то ни каких нарушений не будет.


                          1. vanxant
                            05.10.2018 23:29
                            +1

                            Ну замените поезд на лифт им. Эйнштейна, парящий в межгалактическом пространстве. Нету там никаких потенциалов, ни гравитационных, никаких других. В любой разумной СО потенциальная энергия строго ноль. Телеграфные столбы вдоль дороги считаем невесомыми.
                            У лифта есть двигатель, у двигателя есть мощность, которая и переходит в кинетическую энергию лифта.
                            При этом с точки зрения пассажиров лифта вся наблюдаемая из лифта вселенная ускоряется в обратную сторону.
                            Ещё раз, в неинерциальных системах отсчёта энергия в общем случае не сохраняется. Точка.
                            На этом разрешите откланяться, дальнейшие вопросы прошу адресовать Альберту и Эмми.


                            1. BigBeaver
                              06.10.2018 00:03

                              Откройте для себя пинцип эквивалентности.


                            1. BigBeaver
                              06.10.2018 10:58

                              Сохранение всех трех аддитивных интегралов движения выводится банально из уравнений кинематики при условии однородности и изотропности пространства и времени. Если у СО есть ускорение, это эквивалентно наличию потенциальной силы, и вы просто смело дописываете соответствующее ускорение для каждой элементарной массы в системе, и все продолжает сходиться.

                              Если вы намекаете на неоднородность времени в рамках ТО, то с этим никто и не спорит. Но об этом тут где-то рядом уже написали. Ваш же пример с поездом вовсе не про ускорение, а про якобы различие полной энергии между разными СО.


                              1. vanxant
                                06.10.2018 13:59

                                Поймите простую вещь: я привёл пример с поездом для тех, кто владеет только ньютоновской механикой.
                                Да, его можно исхитриться и «залатать» потенциалами выдуманных полей и прочими математическими трюками. Только это всё равно неверно. Пространство-время в такой системе отсчёта ещё плоское, но уже не изотропное (метрический тензор ещё диагональный и постоянен во всех точках, но на диагоналях уже нет ни одной единицы, из-за чего энергия-импульс не сохраняются).
                                Впрочем, для людей с вашим уровнем подготовки я бы рассмотрел массивный вращающийся шар, на поверхности которого невозможно синхронизировать часы даже локально. На этом наш спор бы закончился.


                                1. Taus
                                  06.10.2018 15:49

                                  Вот вы вначале приводите пример «из жизни», а потом модифицируете его до деталей конкретной теории ускоренного расширения Вселенной, в которой допускается не выполнение законов сохранения. Есть теории, которые объясняют ускоряющееся расширение Вселенной, но законы сохранения выполняются. Будем теперь спорить о том какая теория вернее без экспериментальных проверок?


                                  1. vanxant
                                    06.10.2018 17:53

                                    Простите, расширение вселенной я вообще не трогал. Всё сугубо в рамках ОТО без лямбды.
                                    Также я не трогал, например, исчезновение энергии в черных дырах. Потому что на пальцах принцип «что упало — то пропало» вполне понятен, но объяснить его «на пальцах», не используя тензоры и касательные расслоения, мне например достаточно сложно.


                                1. BigBeaver
                                  06.10.2018 19:22

                                  Я не считаю себя гением ТО, но как ваши слова укладываются в то, что потенциальные и инерциальные силы принципиально неразличимы ни в каком эксперименте?

                                  массивный вращающийся шар, на поверхности которого невозможно синхронизировать часы даже локально.
                                  А на земле-то мы как синхронизируем? Или нам кажется только?


                                  1. vanxant
                                    06.10.2018 23:50

                                    потенциальные и инерциальные силы принципиально неразличимы

                                    Ну это только в самом первом приближении. В реальности все потенциальные поля у нас центральные или близки к центральным. Поэтому при движении возникают эффекты второго и более высоких порядков малости — приливные силы, геодезическая прецессия и т.д., которые вполне обнаружимы (хотя и приходится запускать спутники типа Gravity Probe).


                                    1. BigBeaver
                                      07.10.2018 00:29

                                      центральные или близки к центральным
                                      Между обкладками конденсатора, например (краевыми эффектами пренебречь).

                                      Ну и надо ведь помнить, что мы говорим о вашем «типа бытовом и наглядном» примере с поездом и лифтом.


                                1. BigBeaver
                                  06.10.2018 19:43

                                  Кажется, я понял, в чем у нас проблема. Под «энергия сохраняется» я имел ввиду «выполняется закон сохранения энергии». Но неинерциальная система не замкнута, тк инерциальные силы являются по отношению к ней внешними => энергия сохраняется, но закон сохранения все равно в порядке.

                                  Если вы такой квалифицированный на фоне моего уровня, то почему бы сразу это не сформулировать вместо всяких дурацких примеров?


                        1. Gutt
                          05.10.2018 23:31

                          А неважно. Результирующая скорость системы определится суммой импульсов провзаимодействовавших тел, выделившаяся энергия — деформацией поезда. И абсолютно неважно, врезался поезд в Землю или Земля в поезд.


                1. Taus
                  05.10.2018 19:17

                  При переходе из лабораторной системы отсчёта (СО), когда в начале всё покоилось, в СО связанную с разгоняющимся поездом, то в «законе Ньютона» для Земли добавляется слагаемое с фиктивной силой инерции. Работа всех сил, действующих на Землю, будет в точности равна приобретаемой кинетической энергии Земли в этой СО.


                  1. vanxant
                    05.10.2018 19:49

                    Нет никакой внешней СО, из которой куда-то там переходят с добавлением каких-либо членов.
                    Наш вагон называется «Млечный путь». В окошки телескопов мы видим соседние вагоны-галактики, и вместе с ними мы куда-то едем относительно «пейзажа». Но мы не знаем, есть ли у этого «пейзажа» какая-то небесная твердь, эфир, тёмная энергия или что-то ещё, к чему можно привязать какую-либо «лабораторную СО».
                    Более того, тщательно наблюдая за пейзажем, мы выяснили, что сам пейзаж разбегается от нас с очень маленьким, но таки ускорением. И вот тут-то и возникают пункты статьи номер 7 и 10.


                    1. Zenitchik
                      05.10.2018 21:40

                      Нет никакой внешней СО, из которой куда-то там переходят с добавлением каких-либо членов.

                      Она и не нужна. Слагаемое с фиктивной силой инерции автоматически появляется, когда мы выбираем неИСО.


                1. Iron_69
                  05.10.2018 19:33

                  Ну а если так: паровоз колесами толкает Землю назад, но так как Земля намного тяжелее паровоза, то он движется вперед с ускорением? Т.е. просто посчитать импульсы?


    1. DrSmile
      05.10.2018 16:13

      В ОТО время неоднородно, поэтому теорема Нетер не применима. Однако, уже из самих уравнений ОТО следует локальный закон сохранения энергии, в том смысле, что сколько втекло в объем, настолько в нем и прибавилось. Но из-за динамической геометрии этот локальный закон не дает глобального закона сохранения.

      Однако, тут есть хитрость, что локальный закон сохранения энергии относится только к энергии материи и ничего не говорит про энергию самих гравитационных волн. Если правильно определить энергию гравитационного поля, то можно опять получить глобальный закон сохранения энергии. В случае асимптотически плоской вселенной такая энергия получается достаточно осмысленной. Однако для более сложных конфигураций пространства-времени физического смысла становится гораздо меньше, например, энергию можно произвольно менять выбором системы координат.


      1. Victor_koly
        05.10.2018 23:16

        Гравитационные волны забирают энергию, этот факт давно наблюдается (ну или кто-то другой влияет на скорость движения двойных пульсаров).
        Про непосредственное измерение этого факта можно было бы сказать, если бы мы зарегистрировали энергию грав. волн от ну очень близкого слияния 2 ЧД. Настолько близкого, что мы могли бы астрономически измерить радиусы сингулярностей 2 ЧД до слияния и ЧД после слияния.


    1. denisslvr
      05.10.2018 17:41

      К сожалению не обладаю глубокими знаниями в физике. Но интуитивно, тоже считаю что формулировка не несёт в себе смысла — «и это расширение нарушает закон сохранения энергии.»


      1. denisslvr
        05.10.2018 18:12

        Много вопросов относительно расширения, ну пусть оно есть в результате бв. Энергия в принципе универсальное слово, в общем бульоне, со множеством волн и скоплением частиц (материей) почему вселенная не может быть замкнутой? Почему в ней энергия должна теряться, а не видоизменятся?


        1. LynXzp
          06.10.2018 07:10

          Во-первых расширение увеличивает энергию, а не уменьшает. (Потенциальная энергия)
          Во-вторых расширение вселенной идет не из-за инерции, а за счет темной энергии, энергию которой мы не считаем, поэтому не вижу смысла говорить о нарушении или не нарушении закона сохранения энергии.


          1. denisslvr
            07.10.2018 20:27

            Эм. С чего оно увеличивает энергию?
            Эм. С чего это тёмная материя имеет положительный знак? Т.е. с чего это она является положительной гравитацией которая действует наоборот?


            1. denisslvr
              07.10.2018 20:34

              Мы можем друг — друга не понимать, т.к. я не силён в физике. Но в моём представлении гравитация — это набор атомов и то что они делают, притягиваются. В расширении вселенной прое*бали массу, и ввели тёмную материю, которая неведома, но должна обладать массой, а ещё расталкивать привычную материю, но если она отталкивает, то она должна быть противоположна атому… В общем нужно 0,5. )


              1. LynXzp
                08.10.2018 05:12

                Я тоже не физик, но темная материя и темная энергия никак между собой не связаны кроме сходства в названии. Темная материя это то что позволяет галактикам быть такими какие они есть. А темная энергия это то что увеличивает скорость расширения вселенной.

                Эм. С чего оно увеличивает энергию?

                Правильно ли я понимаю что Вы спрашиваете почему потенциальная энергия тел тем больше чем они дальше друг от друга? (Конечно вне зависимости причин увеличения между ними расстояния.) Надеюсь что нет и Вы просто перепутали темную энергию с гравитационным эффектом темной материи.

                Я немного иронично сказал что расширение вселенной увеличивает потенциальную энергию. Потенциальная энергия как бы увеличивается если увеличивается расстояние между телами, по определению. Но ирония в том что на малом масштабе это увеличение ничтожно мало, а на большом расстоянии ничтожно мало гравитационное взаимодействие, а там и до границы сферы Хаббла недалеко, где вообще нельзя будет рассуждать о потенциальной энергии. И на самом деле, то что я сказал только мое предположение, я могу ошибаться, но все кажется очевидным.


                1. denisslvr
                  08.10.2018 11:05

                  Вы правы, что я неверно в одну корзину положил и тёмную материю и тёмную энергию… Но и то и то является компенсацией массы. Для меня не очевидно что расширение за счёт бв. И для меня не очевидно что есть тёмная энергия. Странно говорить что закон сохранения не работает т.к. галактики удаляются. А при удалении они не теряют массу? А силы которые их «растягивают» ещё и с ускорением не могут быть классическими гравитационными? Моя проблема, что я не в теме. Но очень печально, что существуют тёмные материи и энергии в науке.


  1. AntonSazonov
    05.10.2018 14:16

    Перевод чуть-чуть неточный) В 7-м пункте фигурирует Бруклин, а не Москва))


    1. SLY_G Автор
      05.10.2018 16:16
      +1

      Это локализация.


      1. konst90
        05.10.2018 18:13
        +1

        Но ведь Москва расширяется!


  1. Victor_koly
    05.10.2018 14:23

    Просто так получается, что наблюдаемые нами крупные объекты состоят из множества более мелких, чьё тепловое движение уничтожает все типичные квантовые эффекты.

    Мы можем описать движение частиц по макроскопическим траекториям (вроде движения спутника вокруг планеты) уравнением Шрёдингера. А потом — сделать приближение к уравнению классической механики. Есть ещё промежуточная штука — квазиклассическое приближение. Плохо работает возле точки поворота, то есть не может описать туннелирование.
    А с другой точки зрения — распределения Бозе и Ферми переходят в классическое в области (E-Ef)>>kT.


  1. Oden
    05.10.2018 14:48
    +2

    А я точно все это в школе должен был узнать?


    1. googhalava
      05.10.2018 16:19
      -3

      Да и не в школе зачем мне это узнавать? Никакого прикладного значения для обычного человека ни в одном из 10 нет


      1. vanxant
        05.10.2018 16:28

        А потом такие «обычные люди», даже не осознающие меру своей некомпетентности, составляют КТРУ и гробят экономику государства.


        1. googhalava
          05.10.2018 16:31

          Как это связано с черными дырами и как компьютерная неграмотность связана с энтропией?
          Мне кажется, вы применяете классический прием «Доведение мысли до абсурда».


          1. Zenitchik
            05.10.2018 16:36
            +1

            Да не в связи дело, а в тренировке мозга. Зачем мы изучаем очень много ненужных нам вещей? Затем, что процесс изучения — тренирует мозг, и создаёт условия для изучения нужных вещей.


            1. googhalava
              05.10.2018 16:42

              Статья называется «10 физических фактов, которые вы должны были узнать в школе, но, возможно, не узнали».

              Для тренировки мозга я могу узнать много всего нужного в прикладном смысле.

              И я не берусь назвать какие-то знания, полученные в школе для тренировки мозга, которые мне казались бы более бесполезными, чем эти.

              Кроме того, продолжая эту ветку вы, как бы, поддерживаете точку зрения vanxant по поводу того, что вот без этих уж знаний все и развалится, начиная с Госзакупок.


              1. Zenitchik
                05.10.2018 17:35

                Кроме того, продолжая эту ветку вы, как бы, поддерживаете точку зрения vanxant по поводу того, что вот без этих уж знаний все и развалится, начиная с Госзакупок.

                С чего Вы это взяли? В начале ветки нет такого условия, и никакую галочку об этом я не ставил.


              1. old_gamer
                05.10.2018 18:19

                Кроме того, продолжая эту ветку вы, как бы, поддерживаете точку зрения vanxant по поводу того, что вот без этих уж знаний все и развалится, начиная с Госзакупок.

                Пожалуй, именно в поддержку этого продолжу это ветку, ибо именно это и наблюдаю в данный момент времени.


              1. i86com
                05.10.2018 19:16

                «Да потому, что нам интересно и нравится. А кому не нравится и не интересно, тот, значит, дурак. Без вариантов.»

                Тьфу, блин, как на православный форум попал.

                К слову, в ВУЗе был единственным в группе, кто мог объяснить значение энтропии и интересовался этим. Но без категоричности.


        1. iig
          05.10.2018 16:42

          Вы точно уверены в их некомпетентности в школьной физике?


      1. 0xd34df00d
        06.10.2018 04:30
        -1

        Если выкинуть мои необычные увлечения, то математика мне не нужна совсем (а если не выкидывать, то не нужна лишь школьная математика).

        География мне тоже не нужна совсем. Мне совсем неважно, на каком континенте Килиманджаро, сколько этих континентов вообще, чем тундра отличается от смешанного леса, и что Волга — это не только машина.

        История мне тоже не нужна. Даже как источник уроков. От моего мнения как избирателя все равно ничего не зависит.

        ОБЖ, ну, наверное, нужно.

        Астрономия не нужна точно. К чему мне эти законы Кеплера и пересчитывания звездных величин?

        Физика не нужна. О всем, что нужно, рассказали на ОБЖ.

        Химия не нужна.

        Короче, я походу зря в школу ходил.


        1. 4tlen
          06.10.2018 09:36

          «География? Да извозчики-то на что ж? Только скажи: повези меня туда, — свезут, куда изволишь» Классека


          1. 0xd34df00d
            06.10.2018 16:03

            Именно так. И что самое характерное, извозят.


          1. tvr
            07.10.2018 15:36

            Классека


            русской летиратуры, да.


            1. 4tlen
              07.10.2018 15:54

              Что как бы намекает что невежество и нежелание узнавать новое порицалось уже тогда.


        1. Cast_iron
          06.10.2018 10:43
          +1

          Математика — считаю свои доходы и расходы, стоимость товаров в магазине и пр. повседневные вещи.
          География — сколько времени провел над картой РСФСР изучая места, где были знакомые и родственники, был и буду я сам. На природе определял стороны света и направления.
          История — из уроков я узнал о развитии общества, науки и техники. Да просто расширил кругозор.
          Астрономии у нас не было, а жаль! Мне нравится космос.
          Физика — постоянно использую знания о физических процессах на бытовом уровне — не стоит наливать воду для кипячения до краёв — выльется при расширении, использую рычаги и противовесы, знания об электричестве помогают ковыряться в электроприборах.
          Химия — в быту мало использую, но все же знания о некоторых химических реакциях пригождаются, например: нейтрализация кислоты щелочью.
          Я не понимаю, зачем в меня пихали столько классической литературы, но по большому счету она тоже необходима для изучения вопросов морали и социального поведения.
          И с каждым годом я всё больше убеждаюсь, что в школе не дают ещё множество предметов так необходимых в повседневной жизни.


          1. BigBeaver
            06.10.2018 10:47

            Но если подумать серьезно, все это знает человек, не имеющий образования ни какого вообще. Воду-то в деревнях не умели кипятить, пока физика в школах не появилась, деньги считать не умели, до города соседнего добраться так вообще беда.


            1. Cast_iron
              06.10.2018 10:53

              Только все эти бытовые знания не дают ответа на вопрос «Почему» в отличие от школы.


              1. BigBeaver
                06.10.2018 11:02

                Эти ответы не нужны чтобы пользоваться знаниями. Я вам больше скажу, более 90% людей в школе не знает и не хочет знать этих ваших «почему». Да и вы, когда кипятите воду, вряд ли думаете о физике — просто делаете, как привыкли и все. Я даже не удивлюсь, если окажется, что вы к этому привыкли до того, как началась физика в школе. Я, допустим, еду себе готовил класса с 3, а физика только в 7, кажется.

                upd: и хотя лично я сам очень стремлюсь понимать все эти процессы, это скорее хобби, тк в реальной жизни оно не нужно даже большинству инженеров, а простым смертным и подавно.


                1. Cast_iron
                  06.10.2018 11:45

                  Выше я привел банальные примеры, которыми пользуются часто, но есть и более редкие случаи, которые простыми бытовыми знаниями не обеспечить, не говоря уж о профессиональной деятельности.


                  1. BigBeaver
                    06.10.2018 12:33

                    И я вам показал, что для пользования этим не нужно образование. А ваш собеседние как арз и сказал, что кроме професиональных ему ни какие знания реально не пригождаются.


                    1. 0xd34df00d
                      06.10.2018 16:07

                      Не, ну я с интересом читаю всякие там научно-популярные статьи по физике и биологии, и даже иногда понимаю всякий околофизический хардкор, спасибо образованию. Но это мой интерес и хобби, это не прикладное.


          1. Victor_koly
            06.10.2018 14:03

            Считать доходы без калькулятора — это редко нужны знания после 7 класса.


          1. 0xd34df00d
            06.10.2018 16:02

            считаю свои доходы и расходы

            Табличка в эксельке.


            стоимость товаров в магазине и пр. повседневные вещи.

            Достаточно уметь сравнивать числа.


            постоянно использую знания о физических процессах на бытовом уровне — не стоит наливать воду для кипячения до краёв — выльется при расширении

            У меня на чайнике риска есть, а сколько воды в кастрюлю надо наливать, чтобы потом пенка от пельменей не вылилась, я бы не взялся рассчитывать даже несмотря на то, что в своё время ездил на всерос.


            использую рычаги и противовесы

            Не припомню, когда последний раз их использовал.


            знания об электричестве помогают ковыряться в электроприборах

            Мне пожаробезопасность важнее. Не думаю, что в школе учат физике так, чтобы на эти знания можно было полагаться.


            А если серьёзно, мы тут обсуждаем парадигму «какое прикладное значение имеет $subjname для человека». И ваши расширения кругозоров историей и рассматривания карты РСФСР не сильно отличаются от моих ковыряний нешкольного матана (который, кроме того, помогает и бабло зарабатывать, то есть, имеет значение самое прикладное). И вам этот матан нужен в прикладном смысле примерно так же, как мне карты.


            Ещё довольно забавно, что литература — единственное, что я не стал приводить, предполагая аргумент о моделировании всяких там жизненных ситуаций, а мы ж типа социальные все.


      1. AquiHostStrider
        07.10.2018 12:39

        А не надо узнавать в школе — это ж в 11 классе рассказывают большей частью. 9 классов окончил — и в ПТУ, отыгрывать хомо хабилиса. В самом деле, какая разница, может Земля вообще плоская?


        1. Victor_koly
          07.10.2018 14:48

          И какая разница, если 4-6 лет высшего образования после 11 класса не гарантируют тебе стать Homo ergaster.


        1. googhalava
          07.10.2018 14:50

          Да вот у некоторых людей сарказм и без 9 классов проклёвывается. Далее можно не учиться.


    1. encyclopedist
      05.10.2018 19:12

      Пункты 1,2,3,10 есть в российской школьной программе по физике. Остального вроде нет.


      1. Victor_koly
        05.10.2018 23:25

        Про применимость квантовой механики ничего не помню в школе, т.к. вообще не помню упоминаний о ней. Кажется факт того, что например атом водорода описывается ею уже по химии на 1 курсе учили.
        Закон 2 термодинамики должны были учить, ну а к факту распада частиц его никогда не применял.
        Ну а пункт 10 я все равно трактую наоборот — есть возникающая субстанция с постоянной плотностью энергии и уравнением состояния P = — ro (беру c = 1). И уже она расширяет пространство согласно уравнению Эйнштейна, а не пространство расширяется само.


  1. Anton23
    05.10.2018 15:28
    +1

    Что касается пунктов в статье, то в школе я не про один не слышал.
    P.S. ААААААА, ЭНЕРГИЯ НЕ СОХРАНЯЕЕЕТСЯ!


    1. anonymous
      06.10.2018 12:04

      Ну так Вселенную неправильно считать замкнутой системой. Странно, что в статье об этом не упомянули.


  1. uu_69
    05.10.2018 15:30

    «Причина, по которой тяжёлые частицы распадаются, когда могут, состоит в том, что они это могут.»

    Могут-то они все, а распадаются по очереди. Интересно, каков механизм, запускающий распад данного конкретного атома. Там что, таймер тикает?


    1. Zenitchik
      05.10.2018 15:52

      Интересно, каков механизм, запускающий распад данного конкретного атома.

      В том-то и дело, что нет. Со временем вероятность распада не меняется.


    1. Victor_koly
      05.10.2018 23:49

      Насколько я знаю, альфа-распад хорошо описывается туннелированием. Есть потенциальный барьер (силы, которые делают более выгодными по энергии существование стабильных атомов
      с зарядом ядра до 52 — до теллура-106 нет альфа-распада), а за ним идет яма — альфа-частица отталкивается ЭМ силами от оставшегося ядра.
      Может за счет 3-мерности задачи выйдет показать, что альфа-частица с высокой вероятностью оказывается за этим барьером. Почему у самых стабильных изотопов (например урана беру, а не висмута) частица задерживается в ядре 23 миллиона, 704 миллиона или 4.5 миллиардов лет — это я точно не знаю.
      В ещё может так быть, что с вероятностью 1/1000 частица распадается немного не так — с рождением пары более тяжелых кварков. А потом одна из родившихся частиц через 0.15 нс скорее всего распадается. Но в любом случае энергия покоя 2 новых частиц меньше, чем энергия распавшейся.


  1. Igor_O
    05.10.2018 15:55

    А пункту 5 я один удивился? Гугл находит больше десятка тысяч научных публикаций про способы прикрутить квантовую запутанность к передаче и обработке информации, и вдруг «их нельзя использовать для передачи сообщений»…


    1. Shkaff
      05.10.2018 16:17
      +1

      То, что ее можно «прикрутить» — т.е. использовать, например, для шифрования, не значит, что с ее помощью можно передать информацию. Нельзя взять пару запутанных частиц, дать одну Алисе, другую Бобу, и, сделав что-то с одной частицей, передать бит информации от Алисы к Бобу. Называется «no-communication theorem» .


      1. Igor_O
        05.10.2018 16:56

        Не углубляясь в математику процесса, но если мы придумаем детектор того, является ли состояние частицы детерминированным или нет, мы получим передачу информации. Простой вариант — берем пару запутанных фотонов. Один фотон пропускаем через поляризационный фильтр (или не пропускаем), а его пару на минимальный отсчет времени позже проверяем (методом, например, тех самых «слабых» квантовых измерений из комментария ниже), поляризован фотон или находится в неопределенном состоянии. И получаем передачу бита.
        И да, «прикрутить» шифрование — вообще отдельный интересный вопрос. Передаем ли мы передавая зашифрованное сообщение дополнительную информацию по сравнению с тем же незашифрованным сообщением? Ведь если и отправитель и получатель знают незашифрованное сообщение, передача зашифрованного сообщения передает всю необходимую информацию о ключе шифрования, который сам может быть сообщением.
        (PS: кстати, шифр, использовавшийся посольствами Германии во время второй мировой взломали исходя из гораздо меньшего объема информации. Была ошибка при приеме сообщения в Германии, запросили повтор. Повторное сообщение было отправлено. Шифр был аналогичен Энигме и использовал поворачивающиеся колеса. Перехватившие сообщение агличане в лице того самого Алана Тьюринга зная, что зашифровано одно и то же сообщение, высчитали количество и взаимодействие колес в шифровальной машине, т.е. взломали шифр...)


        1. Shkaff
          05.10.2018 17:05
          +2

          Первый момент: любая запутанность фундаментально случайна (об этом как раз в статье). Нет никакого способа узнать, получаете ли вы случайную последовательность или «запутанность», пока вы не сравните результаты измерения у Алисы и Боба. Измерение только у Боба выглядит просто как случайный шум, только корреляция с Алисой позволит найти запутанность. Не существует способа проверить «находится ли фотон в неопределенном состоянии».
          Второй момент: про ваш вариант. Допустим, вы запутали HV поляризации фотонов. Вы измерили один, например, в H — второй коллапсировал в V. Фотоны к вам приходят в случайной поляризации, так что вы измеряете 01101110010. Второй фотон, соответственно, в 10010001101. Каким образом эти две последовательности могут являться «битом» информации? Если вы сравните — увидите идеальную корреляцию, как и положено, но так то сравнить надо…


          1. Igor_O
            05.10.2018 18:34
            -1

            Тут как раз штука в том, что все еще достаточно много ученых пытаются найти способ «неразрушающего контроля» коллапсировала волновая функция или нет. Если способ будет найден — то просто ставим один за другим управляемый поляризационный фильтр и поляризационный фильтр. На приемнике ставим два последовательных детектора детерминированности состояния. Если на первом состояние детерминирвано — все просто, пришла единица. Если на первом недетерминировано, а на втором детерминировано — пришел нолик. Если на первом и втором недетерминировано, отбрасываем, не запутанный фотон.


            1. Shkaff
              05.10.2018 18:48
              +1

              Тут как раз штука в том, что все еще достаточно много ученых пытаются найти способ «неразрушающего контроля» коллапсировала волновая функция или нет.

              А можно ссылочку?
              Потому что с моей колокольни это утверждение просто не имеет смысла. Запутанность по определению завязана на корреляциях между двумя частицами, и глядя на одну частицу никак-никак нельзя понять что она «запутанна».

              Если вы говорите о слабых измерениях и тп — это не про то. Вы не сможете узнать состояние одной частицы без набора статистики.

              Кстати, коллапс волновой функции — это скорее способ производить вычисление, а не обязательно нечто физическое. Есть множество интерпретаций квантов, которые обходятся без коллапса.


              1. Igor_O
                05.10.2018 20:47

                Кстати, коллапс волновой функции — это скорее способ производить вычисление

                Совершенно верно. Вся квантовая физика — это способ производить вычисление. Но вот почему-то светодиоды — светят, а транзисторы — усиливают сигнал, хотя основаны на чисто математических феноменах…

                А можно ссылочку?

                www.google.com не подходит чем-то? «wave function collapse detection» для начала. Уже дает несколько интересных ссылок. Дальше можно уточнять по ключевым словам.

                Запутанность по определению

                «квантовомеханическое явление, при котором квантовые состояния двух или большего числа объектов оказываются взаимозависимыми.»
                … никто не может правильно это объяснить. Я, вот, как и множество исследователей, понимаю запутанность не как «статистическую корреляцию», а как прямую связь между состояниями запутанных частиц. И, как я уже говорил, несмотря на то, что в википедии написано, что информация при этом не передается и «теория относительности не нарушается», но вот каким-то образом информация о том, что мы с одной из запутанных частиц что-то сделали, передается со скоростью, превышающей скорость света. Всего-то осталось изобрести детектор этого факта и получим передачу информации со скоростью выше скорости света.

                лядя на одну частицу никак-никак нельзя понять что она «запутанна»

                Если нафантазировать, что мы изобрели детектор коллапсированности волновой функции, не коллапсирующий волновую функцию, я чуть выше описал простой способ передачи информации и отсеивания не запутанных частиц.


                1. Shkaff
                  05.10.2018 20:56
                  +3

                  www.google.com не подходит чем-то?

                  Тем, что если мы обсуждаем физику, правила хорошего тона предписывают давать ссылки на конкретные источники.
                  «wave function collapse detection» для начала. Уже дает несколько интересных ссылок.

                  Ну и при чем тут это? Это не имеет никакого отношения к обсуждению.
                  но вот каким-то образом информация о том, что мы с одной из запутанных частиц что-то сделали, передается со скоростью, превышающей скорость света.

                  Ну, то, что вы чего-то не понимаете, не значит, что это природа обязана подчиняться рамкам вашего понимания. Ваше утверждение просто неверно.
                  Если нафантазировать,

                  Нафантазировать вы можете что угодно, но к физике это отношения не имеет. Фэнтези же мне не очень интересно обсуждать.


                  1. Igor_O
                    05.10.2018 23:21

                    Нафантазировать вы можете что угодно, но к физике это отношения не имеет. Фэнтези же мне не очень интересно обсуждать.

                    Тогда зачем вы обсуждаете квантовую физику? Примерно вся квантовая физика — математические фантазии. Как вы пронаблюдаете в реальном мире корень из -1? А без него просто нет никакой квантовой физики.

                    Ну и при чем тут это?

                    Например при том, что там есть несколько ссылок на научные работы. Просто сам факт наличия таких научных работ говорит о том, что пока у некоторых физиков все еще есть надежда, что удастся засекать коллапс волновой функции отдельно от измерения, которое приводит к коллапсу волновой функции.
                    Но с учетом работ А. Короткова и Э. Джордана про «слабые квантовые измерения», все еще проще. «Приемник» ставим дальше «передатчика» от источника спутанных фотонов. Для каждого фотона в «передатчике» производятся «слабые измерения» до тех пор, пока не получим необходимое состояние для 0 или 1. В этот момент «фиксируем» результат измерения, получаем необходимый бит на «приемнике».


                    1. Shkaff
                      06.10.2018 09:23

                      Тогда зачем вы обсуждаете квантовую физику?

                      Потому что квантовая физика — моя специальность, я за это деньги получаю, и мне интересно обсуждать спорные моменты. Но не чужие фантазии.
                      И да, если вы мне дадите математическую основу ваших фантазий, у нас пойдет совсем другой разговор. Потому что ваши фантазии сейчас не основаны ни на физике, ни на математике. Поэтому обсуждать их бессмысленно.
                      Например при том, что там есть несколько ссылок на научные работы.

                      А вы читали эта работы-то? Думаю, нет. А я читал, и они не имеют отношения к обсуждаемому.
                      Но с учетом работ А. Короткова и Э. Джордана про «слабые квантовые измерения», все еще проще.

                      Слабые измерения работают не так. Слабое измерение значит, что вы очень слабо воздействуете на объект, но и не получаете при этом никакой информации почти. Так что получить ваше «необходимое состояние» просто невозможно.


                1. BigBeaver
                  05.10.2018 22:13

                  но вот каким-то образом информация о том, что мы с одной из запутанных частиц что-то сделали, передается со скоростью, превышающей скорость света.
                  А кто вам сказал, что она вообще передаетсяя? И нет, речь не о концеции скрытых параметров.


                1. kauri_39
                  05.10.2018 22:30
                  -2

                  «каким-то образом информация о том, что мы с одной из запутанных частиц что-то сделали, передается со скоростью, превышающей скорость света»

                  Да, опыты около Женевы в 2008 году подтвердили, что скорость взаимодействия запутанных фотонов как минимум в 100 000 раз превышает скорость света. Игорь Иванов об этом хорошо написал.
                  Вот и надо разбираться — «каким образом» запутанные частицы постоянно и мгновенно обмениваются информацией о состоянии своего спина, чтобы при всех его переменах общий спин их квантовой системы (пары частиц) был равен нулю. Каждая из частиц тут же меняет свой спин на противоположный, как только получит сигнал об изменении спина у своей напарницы. Возможно, что носителями сигнала служат продольные волны в плотном вакууме.
                  Поскольку без измерения нельзя понять, какой спин у любой из частиц, считают, что у частиц суперпозиция спинов. Такая математическая условность не отражает реальность, огрубляет её, делает вероятностной. Это почти как строить теории мультиверса, не имея возможности их проверить.


                  1. Fracta1L
                    06.10.2018 08:40

                    Такая математическая условность не отражает реальность, огрубляет её, делает вероятностной

                    Пичалька, да? Не укладывается в мышление 18 века, да? XD


                    1. kauri_39
                      06.10.2018 10:08

                      Физика 20 века и не должна укладываться в мышление 18 века. А новая физика 21 века не будет укладываться в мышление 20 века, это нормально. Печально то, что моё окружение продолжает мыслить прошлым веком.


                      1. Fracta1L
                        06.10.2018 13:02
                        +2

                        А новая физика 21 века не будет укладываться в мышление 20 века

                        Но будет в мышление 18-го, я понял.


          1. Victor_koly
            05.10.2018 23:56

            Для шифрования это вполне годится. Сначала создается пара ключей. Потом Алиса шифрует измеренным ключом 01101110010 свою инфу. Тут главное договориться, чтобы Боб не попробовал узнать свой ключ до измерения Алисы, но не будет же он узнавать ключ до шифрования?
            Потом Боб из измеренного ключа 10010001101 получает исходный ключ Алисы и расшифровывает послание.


            1. Shkaff
              06.10.2018 09:25

              Ну да, именно так! Про то и речь, что для шифрования — очень полезно. Но речь-то про передачу информации, а вот с этим тяжко…


      1. SilverHorse
        06.10.2018 16:36
        +2

        Кстати, раз уж в комментах наш главный физик Хабра: Shkaff, вам не кажется, что в п.2 в статье используется подмена понятия «масштаб применимости» понятием «расстояние»? Да, мы можем разнести запутанные частицы на миллиард километров и наблюдать их, да, они будут демонстрировать те же свойства… но они от этого объектами макромира со всеми вытекающими не станут же.

        Да и вообще в статье перечислены факты, которые являются коллекцией непониманий определений или подменой понятий. Мне особенно понравился п.4 — это, видимо, для тех людей, которые диаграммы Фейнмана видели где-то на картинках, но ни разу не пытались ими пользоваться или уяснить для себя, что именно они означают и как их читать. То же самое про п.8 — червоточины это не научная фантастика, это математический объект, возникающий при определенном решении уравнений ОТО. Равно как и черная дыра является такой же математической абстракцией, а вот коллапсар, который порождает область пространства-времени, которая этой абстракцией описывается, вполне реален. Научной фантастикой на данный момент являются не червоточины, а гипотетические объекты, которые их могут создавать, пока не найдены хотя бы косвенные свидетельства возможности их существования.

        И еще поясните п.7 — я что-то путаю, или на самом деле все прекрасно расширяется, просто 1) масштаб расширения ничтожно мал, 2) расширяются не атомы, а пространство, которое их вмещает, причем расширяется изотропно, и с нашей точки зрения относительные расстояния, то есть интервалы, не меняются никаким образом? Иными словами, метр для нас остается метром, просто сам этот «метр» медленно, но изменяется (и заметить мы это сможем, только когда расширение достигнет той стадии, когда будет наблюдаемо на локальных масштабах нашего мира, в последние моменты перед Большим Разрывом)?


        1. Shkaff
          06.10.2018 17:29
          +1

          вам не кажется, что в п.2 в статье используется подмена понятия «масштаб применимости» понятием «расстояние»?

          Я думаю, просто автор жертвует ясностью в пользую краткости. Первое утверждение верно: в стандартной квантовой физике нет никаких ограничений на размеры/массы описываемых объектов. По идее нет причин полагать, что существует граница между квантовой и классической физикой, просто упомянутый процесс декогеренции квантовой системы приводит к тому, что мы наблюдаем большие объекты классически. Сейчас работают над экспериментами, где массивные объекты (у нас в группе, например, 100 граммовые зеркала), приготавливаются в запутанном состоянии.

          Да и вообще в статье перечислены факты, которые являются коллекцией непониманий определений или подменой понятий.

          Согласен, не лучшая статья Сабины, прямо скажем. Плюс перевод, как обычно, далек от идеала…

          И еще поясните п.7

          В моем представлении в статье все правильно. Во-первых, расширение можно рассматривать просто как силу, действующую на атом, и растягивающую его в стороны. Эта сила просто слишком мала, чтобы хоть как-то повлиять на размер.
          Во-вторых, тут важный момент, что расширение не абсолютно, а зависит от самого объекта. Если объект связан гравитационно (как солнечная система) или атомно (как атом), то и расширение практически не будет иметь эффекта на него. Если взять обычную скорость расширения и пересчитать ее на атом, эффект будет более значительным. Можно вот тут почитать хорошую старую статью: arxiv.org/pdf/astro-ph/9803097v1.pdf или вот тут поновее.


          1. kauri_39
            07.10.2018 15:15
            -3

            «расширение можно рассматривать просто как силу, действующую на атом, и растягивающую его в стороны. Эта сила просто слишком мала, чтобы хоть как-то повлиять на размер.»

            А сила, действующая на скопления галактик, достаточна велика, чтобы их раздвигать с ускорением. Но если расширяющееся пространство обладает движущей силой, то как рассматривать само пространство? Очевидно, как энергетически плотную среду. Причём как очень плотную, если она с тем же ускорением гонит друг от друга скопления разных масс и концентраций материи.

            «массивные объекты (у нас в группе, например, 100 граммовые зеркала), приготавливаются в запутанном состоянии.»

            Поясните, пожалуйста, каким образом эти зеркала будут проявлять своё запутанное состояние?


            1. Shkaff
              07.10.2018 16:14
              +1

              Да-да, мы знаем, что вы эфирщик:)

              Поясните, пожалуйста, каким образом эти зеркала будут проявлять своё запутанное состояние?

              В корреляциях между их положениями и импульсами, которые сильнее, чем классические. По сути, это прямая реализация ЭПР парадокса. Например, измерив расстояние между ними, мы можем предсказать разность импульсов с точностью лучше, чем дана принципом неопределенности (в идеальной системе абсолютно точно).


              1. kauri_39
                07.10.2018 21:44
                -2

                Лишь бы мой эфир вписывался в наблюдения, а теория подтянется…

                Но у вас в группе фантастики больше. Как я понял, вы не просто готовитесь обойти неопределённость Гейзенберга, а превысить скорость света в передаче информации. Меняя положение одного зеркала, вы не только можете предсказать новое положение запутанного с ним зеркала, но и тут же проверить своё предсказание, например, по смещению на экране пятна отражённого им света. Если корреляция действительно есть, то она будет сохраняться вне зависимости от расстояния между зеркалами. Впрочем, если признание в этом нарушает подписку о неразглашении, то фрик-эфирщик, как всегда, всё понял неправильно :)


                1. Comod
                  07.10.2018 22:13

                  Лишь бы мой эфир вписывался в наблюдения, а теория подтянется…

                  Интересно, темная материя имеет какое-то отношение к вашему эфиру? И почему у концепции от которой в свое время отказался даже ее создатель, развелось столько апологетов в современном РУнете?


                  1. kauri_39
                    08.10.2018 09:26
                    -1

                    Начнём с того, что термином эфир с давних пор обозначают среду, заполняющую всё пространство. То есть пространство является средой. Пространство-среда является логической противоположностью пространства-пустоты. Пустота по латыни называется вакуум.
                    Проявление эфира искали в виде горизонтального эфирного ветра. Он должен был возникать при движении Земли сквозь неподвижный эфир. Но такого эфира не нашли и вместо него ввели понятие вакуум — пространство-пустоту.
                    Однако эта пустота влияла на поведение частиц — вызывала их Лэмбовский сдвиг. Поэтому в микромире вернулись к понятию среды, но чтобы не ссориться с физиками макромира назвали её физическим вакуумом. Потом обнаружили, что и в макромире пустота влияет на поведение материи — с ускорением раздвигает скопления галактик. Но в силу инерции её продолжили называть вакуумом, иногда добавляя к этому термину прилагательное «космический». Типа это всё же не пустота, а такая среда, с не очень большой плотностью, зато — с постоянной (космологической), которая почему-то не убывает с расширением пространства. Типа есть внутри её какая-то тёмная энергия.
                    Я уже делаю скидку на человеческие недостатки в физическом сообществе и вместе с ним называю эфир, среду вакуумом. Есть же пословица — «хоть горшком называй, только в печь не ставь». Мы же и атомы называем «атомами» — неделимыми частицами, хотя узнали, что они делимые, и поэтому должны называться «томы». Привыкли, и ладно. Но, видимо, не весь современный РУнет такой покладистый — в отношение перемены в названии среды. Ведь эфир переименовали в вакуум уже после рождения науки, а наука — не религия и должна признавать свои ошибки, называть вещи своими именами. И апологеты эфира не все «белые и пушистые», их эфир часто образца позапрошлого века, далёкого от реальности.

                    Плотная расширяющаяся среда устраняет потребность в тёмной материи, но об этом я много писал здесь, почитайте мои прошлые комментарии.


                1. Shkaff
                  07.10.2018 22:19
                  +2

                  Как я понял, вы не просто готовитесь обойти неопределённость Гейзенберга, а превысить скорость света в передаче информации.

                  Никакая информация не передается.
                  Меняя положение одного зеркала, вы не только можете предсказать новое положение запутанного с ним зеркала, но и тут же проверить своё предсказание, например, по смещению на экране пятна отражённого им света.

                  Конечно, мы не меняем сами положение зеркал — это бы разрушило корреляции. Мы собираем информацию о положении/моменте двух зеркал, а потом эта информация коррелируется друг с другом. Никакой информации быстрее скорости света между зеркалами не передается, в этом суть нелокальности квантового мира.


                  1. kauri_39
                    08.10.2018 00:15
                    -1

                    Я понял: никакая информация не передаётся.

                    Теперь, если можно, о деталях. Запутанность зеркал проявляется в корреляции их положений, но сами вы не меняете положение зеркал, чтобы не разрушать этой корреляции. Вы просто собираете информацию о переменах в положении/моменте двух зеркал по 100 грамм каждое, а потом сравниваете, какой уровень корреляции (и как его повысить, чтобы приблизиться к идеальной системе).
                    Это очень умное изобретение. Если нельзя уменьшить инструмент измерения — фотон, то, чтобы обойти неопределённость и вызванную измерением декогеренцию, надо увеличить объект измерения — перейти от запутанных частиц к макрообъектам. Лишь бы научиться запутывать макрообъекты. Если получается запутать 100 граммовые зеркала, то измерять их положения можно даже не одним фотоном, а целым пучком фотонов — по отражаемому ими свету, да?
                    Представляю, как лазерный луч светит на одно из запутанных зеркал, подвешенное на крутильных весах, реагирующих на давление света. Другое зеркало удалено пока лишь в противоположный конец лаборатории и тоже отражает свет своего лазера, отклонившись на весах под его давлением. И кто-то (не вы сами, не из вашей группы) в нужной ему последовательности отключает и включает первый лазер. А вы только регистрируете изменения в положениях обоих зеркал — пусть не совсем совпадающие (система не идеальна), но синхронные (в силу нелокальности квантового мира). Фантастика?


                    1. Shkaff
                      08.10.2018 08:57

                      Через пару лет — реальность:)

                      измерять их положения можно даже не одним фотоном, а целым пучком фотонов — по отражаемому ими свету, да

                      Да
                      другое зеркало удалено пока лишь в противоположный конец лаборатории

                      Кстати нет, нам не интересно проверять нелокальность — зеркала находятся близко друг от друга. Так, что их можно пододвинуть достаточно близко, чтобы взаимная гравитация стала значительной и (возможно) разрушала запутанность.
                      кто-то (не вы сами, не из вашей группы) в нужной ему последовательности отключает и включает первый лазер.

                      Зеркала измеряются одним и тем же лазером, мы измеряем их относительное движение (в интерферометре). Не понял, к чему этот другой человек.


    1. vanxant
      05.10.2018 16:21

      Речь не про запутанность как таковую, а про мгновенность передачи.
      Те статьи, что вы нагуглили, они про то, что вот мы взяли две запутанных частицы, одну себе оставили, вторую отправили корреспонденту по классическому каналу связи (со скоростью не больше скорости света). Смысл же в том, что измерить такую частицу можно только один раз, после чего запутанность разрушается. Если даже товарищ Ева перехватит отправленную нами частицу, она может её только испортить, но не прочитать (для чтения нужно знать «ключ шифрования» — т.е. базис, в котором производилось запутывание).
      PS. На самом деле, есть некоторый хайп насчёт слабых (обратимых) квантовых измерений. Идея там в том, чтобы проводить «запись» информации уже после того, как вторая частица отправлена получателю. Т.е. конверт с письмом вы отправляете обычной почтой, чтобы не сердить дедушку Эйнштейна, но вот буквы в письме появляются уже в процессе его доставки.


      1. Shkaff
        05.10.2018 16:33

        На самом деле, есть некоторый хайп насчёт слабых (обратимых) квантовых измерений.

        А что такого хайпового-то? Тут их часто поминают, но мне кажется, есть какое-то недопонимание (или у меня, или у других). По мне так это же обычные квантовые измерения, и никакой особенно способности к передаче информации у них нет.

        Кстати, слабые не равно обратимые. Есть три понятия: weak measurement, weak value, quantum eraser, и они связаны, он не всегда и не обязательно.


  1. crazy_mama
    05.10.2018 16:23

    Это такое в школе объясняют?! 0_о


    1. vanxant
      05.10.2018 16:47

      Ну, 1, 3 и 10 таки в школе. Остальное — в высшей школе, которая в английском таки тоже school.


      1. Zenitchik
        05.10.2018 17:38

        Простите, что? В курсе какого школьного предмета есть энтропия? Можете сослаться на учебник?


        1. vanxant
          05.10.2018 17:49

          В курсе физики, конкретно в той части, где про молекулярную физику. Ну там где про идеальный газ, степени свободы в зависимости от количества атомов в молекуле и вот это всё. Там совершенно точно рассматривался газ в сосуде, разделённом пополам мысленной стенкой. И где каждая молекула может быть слева или справа с вероятностью 50%, но ситуация, когда они все собрались только с одной стороны, математически невероятна. И из этого выводился закон Паскаля (про то что газ давит равномерно во все стороны).


        1. sergey-gornostaev
          05.10.2018 18:39
          +1

          Если мне не изменяет память, термодинамику нам преподавали в 10-м классе. И если не ошибаюсь, по такому учебнику
          image


          1. Zenitchik
            05.10.2018 21:49

            Да, в этом учебнике рассказывается о необратимости процессов, о хаосе и иже с ним, о вероятности различных состояний — короче, обо всём том, что говорят, когда объясняют энтропию, но сам термин «энтропия» не даётся.


            1. sergey-gornostaev
              06.10.2018 09:36

              Преподаватель использовать термин «энтропия», когда давал главу. Впрочем, воспоминания об этом у меня несколько смазаны, так как я к тому моменту уже года три как был знаком с этой темой.


              1. Zenitchik
                06.10.2018 14:31

                Преподаватель использовать термин «энтропия», когда давал главу

                Вам повезло. Вы — исключение. Я термин «энтропия» узнал сам, а если бы не узнал — то впервые столкнулся бы с ним на втором курсе на физхимии, вместе с энтальпией и свободной энергией Гиббса.


                1. antanariva
                  06.10.2018 16:20

                  Мне в школе в 10 или 11 классе преподавали энтропию именно как «мера беспорядка». Термин был, а задач с ним — нет. Ничего не запомнилось.


                  1. Zenitchik
                    06.10.2018 16:38

                    А у меня — на втором курсе — были задачи. И из этих задач никакого беспорядка не следовало, поэтому я запомнил, что энтропия вряд ли связана с чем-то наблюдаемым и нужна только для, как промежуточная величина.


    1. Comod
      05.10.2018 19:43

      Я тоже удивлен, в школе то и СТО мало где проходят, только в физ-мат лицеях и крайне поверхностно, а тут такое нагнала дама, черные дыры, расширение пространства..., сума сойти!)


      1. sergey-gornostaev
        06.10.2018 09:38

        Про чёрные дыры и расширение вселенной нам рассказывали на астрономии.


  1. maslyaev
    05.10.2018 17:41

    Капелька оффтопа

    Энергия сохраняется.
    Что интересно, деньги тоже. То есть выражение «общество не может себе позволить тратить слишком много денег на благотворительность» не имеет смысла по той же причине, по которой система не может минимизировать свою энергию.


    1. old_gamer
      05.10.2018 18:26
      +2

      Деньги сохраняются только в инерциальной системе отсчета экономике, лишенной мультипликатора


  1. QtRoS
    05.10.2018 18:51

    Про черную дыру же обычно наоборот пишут, что для тебя это будет бесконечно долгое падение, а издалека если смотреть, то тебя растянет и засосет. Где истина?
    (равда в публикациях по теме часто полно противоречий и нерешённых загадок)
    (хотя судя по вики все будет как в статье)


    1. vanxant
      05.10.2018 19:29

      Нет, если ЧД достаточно большая, вы в неё очень быстро (по своим часам) упадёте, причём момент пересечения горизонта ЧД можете даже не заметить. А для внешних наблюдателей вы просто исчезнете в красном смещении (мощность любых сигналов, какие бы вы не посылали наружу, довольно быстро падает и становится ниже мощности космических шумов).
      Вообще, пункт 8 самый слабый из всех и, имхо, вообще ни о чём. И ЧД, и кротовые норы — это решения уравнений Эйнштейна для некоторых, вполне распространённых в космосе начальных условий. Если мы считаем математику наукой, то и то, и другое — стопроцентная наука. Дальше в разделе 8 идут вообще какие-то сопли. Да, гравитационные линзы — но не только ЧД работают линзами. Солнце — не ЧД, но именно наблюдения за гравитационной линзой Солнца убедили физиков в верности ОТО. Да, компактные объекты, но не все компактные объекты — ЧД. Да, у ЧД специфическое излучение, но даже если мы бы могли его зарегистрировать, мы не смогли бы по этому излучению отличить ЧД от просто очень холодного камня или капли жидкого гелия. Это всё очень косвенные признаки, которые не отрицают существования ЧД, но ни в коем случае не доказывают его.
      С чревоточинами, правда, ещё хуже — мы даже теоретически не знаем, как поставить эксперимент и увидеть хотя бы косвенные признаки их реального существования в нашей Вселенной.
      О чём нужно было написать в разделе 8 — так это о том, что гравитационный телескоп ЛИГО действительно увидел именно ЧД. Точнее, звон от их слияния. Уж очень он характерный. Если бы сливались нейтронные звёзды — был бы мощнейший взрыв (и позднее мы такое увидели), какие-нибудь неведомые квантовые звёзды могли бы, например, слиться неупруго с образованием «груши», но мы увидели именно звон дрожащего от слияния горизонта объединённой ЧД.


      1. Victor_koly
        06.10.2018 00:11

        Теоретически мы могли бы видеть линзирование на ЧД. Но это нужно точно измерить такие факты:
        1. Мы видим искривление луча света. Как пример — «крест Эйнштейна» с явно очень похожими спектрами изображений. Ну и чтобы не вышло, что там разница времени пути 10 тысяч лет, звезда имеет полный срок жизни всего 1 миллион и сильно изменится спектр за это время.
        2. Мы посчитали необходимую массу M гравитирующего тела и точно посчитали, что изменение траектории из пункта 1 не может быть вызвано массой, сосредоточенной в радиусе более 2GM/(c^2). Скажем при большем радиусе у нас слишком большая часть света по траектории падала бы на звезду.

        Но вообще-то выходит бред наверное. Но можете посчитать, какой радиус при плотности нейтронной звезды будет иметь «звезда» массой 4 миллиона масс Солнца. Или условная «плотность» такой массивной ЧД уже меньше?


      1. Comod
        06.10.2018 15:42

        причём момент пересечения горизонта ЧД можете даже не заметить.

        Нет вы просто не доживете до того момента, так как вас разорвет градиентом силы на точечные частицы.


        1. vanxant
          06.10.2018 15:45
          +2

          Для достаточно крупной ЧД (например, в центре галактики) — нет.


          1. Comod
            06.10.2018 16:03

            Да уж нет, при приближении к ГС для любой ЧД сила стремится к бесконечности: F=GM/(R^2*(1-Rs/R)^(1/2)).


            1. vanxant
              06.10.2018 16:07
              +1

              Важна не сила, а её градиент. Сама по себе сила действует на все частицы объекта.


              1. Comod
                06.10.2018 20:35
                -2

                Градиент будет в любом случае при приближении к ГС, вопрос лишь в том на каком относительном расстоянии от центра масс(R/Rs) вас разорвет в клочья. Но это еще не конец, как вы могли читать мой пост пониже, гравитация это неоднородность времени, так вот горизонт событий это та граница за которой интервал любых часов хоть покоящихся хоть не покоящихся будет равен 0, а это значит отсутствие времени как такового вместе с пространством и энергией и тогда за ГС никаких часов не может быть, а Чд в таком случае представляет область пространства-времени без пространства-времени. По этому я считаю, что совместное существование горизонта событий и сингулярности не возможно в принципе, может быть только одно из них и я больше склоняюсь к сингулярности.


                1. 4tlen
                  06.10.2018 20:51
                  +1

                  То что считаете Вы имеет отношение к науке?

                  я считаю, что совместное существование горизонта событий и сингулярности не возможно в принципе
                  ГС и сингулярность это решения уравнений ж. У вас есть что противопоставить им?


                  1. Comod
                    06.10.2018 21:06
                    -2

                    Я в курсе про метрику Шварцшильда, но ее выводы противоречат логике и причинно следственным связям. Если вы не согласны, то будьте добры объяснить какую полную энергию будет иметь частица или фотон при пересечении ГС, если она начала падать с такого то расстояния «R/Rs» и имело начальную энергию «Ео» Если в итоге у вас получится не 0 и не бесконечность, то поздравляю вы доказали, что черных дыр, такими какими мы их знаем, не существует.


                    1. 4tlen
                      06.10.2018 22:41

                      но ее выводы противоречат логике
                      чей логике? К слову логика вообще не наука.
                      какую полную энергию будет иметь частица или фотон при пересечении ГС
                      вот этого вообще никто не скажет вам


                    1. BigBeaver
                      06.10.2018 22:57
                      +1

                      Просто признайтесь, что вы называете логикой бытовую интуицию. Математические выводы принципиально не могут противоречить логике, к слову. Если исходные посылки верны, то и решения верны.


                      1. Comod
                        06.10.2018 23:11
                        -1

                        Логика состоит в том, что гравитационный потенциал, гравитационное красное смещение и гравитационное замедление времени эквивалентны при любых условиях в том числе в условиях черной дыры:
                        Фи=mc^2*Zg(red) = mc^2*(Tay1/Tay2-1) по этому считать ее бытовой логикой, частным случаем или приближением вы не можете, точка. Случай когда Tay1/Tay2=0 это либо горизонт событий, либо сингулярность, а в итоге мы получаем -mc^2, где минус как бы намекает на отдачу энергии, а не на ее поглощение. Если думать дальше то выходит, что масса черной дыры имеет:

                        Также для их наличия требуется отрицательная энергия, которую никто никогда не видел, и по поводу существования которой у многих физиков есть большие сомнения.

                        она самая — отрицательная энергия, только без червоточины.


                    1. vanxant
                      06.10.2018 23:45

                      будьте добры объяснить какую полную энергию будет иметь частица или фотон при пересечении ГС, если она начала падать с такого то расстояния «R/Rs» и имело начальную энергию «Ео»

                      Понятие «энергия частицы» имеет смысл (конкретное числовое значение) только в какой-либо конкретной системе отсчёта. При переходе в другую СО, например, движущуюся относительно первой, энергию нужно пересчитывать. Надеюсь, вы не будете с этим спорить?
                      Так вот, в своей собственной СО частица имеет ровно столько энергии, сколько ей предписывает иметь квантовая механика.
                      Метрика Шварцшильда и её производные, ну всякие там Керры-Ньюманы, вам не нравятся.
                      Никакую наружную (относительно ЧД) метрику нельзя продлить до пересечения с горизонтом событий. Как бы вы не изворачивались, временнАя координата частицы в момент пересечения ГС будет лежать на бесконечности, т.е. вне вашей наружной метрики. Поэтому говорят, что ЧД — это область, выколотая из нашей Вселенной. Соответственно и приписать частице какую-либо энергию в такой метрике при пересечении ГС нельзя.
                      Я ответил на ваш вопрос?


                      1. Comod
                        07.10.2018 01:14

                        Понятие «энергия частицы» имеет смысл (конкретное числовое значение) только в какой-либо конкретной системе отсчёта.
                        При переходе в другую СО, например, движущуюся относительно первой, энергию нужно пересчитывать. Надеюсь, вы не будете с этим спорить?

                        Только там полная* энергия частицы — mc^2*y. А гамма будет в точности равно различию интервалов времени в разных СО, вызванных гравитационным замедлением времени, если частица в начале «покоилась».
                        Так вот, в своей собственной СО частица имеет ровно столько энергии, сколько ей предписывает иметь квантовая механика.

                        А вот здесь не понятно, во первых к чему вы упомянули квантовую механику? И во вторых собственная СО это немного не корректное определение, так как собственная это та СО, которая связана с движущимися часами падающей частицы на которую кроме гравитационных релятивистских эффектов еще влияют обычны эффекты СТО. Нужно говорить о гипотетической координатной СО, которую в определенный момент пересечет падающая частица имея определенную для этой координаты энергию, импульс и соответствующую им скорость.
                        Никакую наружную (относительно ЧД) метрику нельзя продлить до пересечения с горизонтом событий. Как бы вы не изворачивались, временнАя координата частицы в момент пересечения ГС будет лежать на бесконечности, т.е. вне вашей наружной метрики. Поэтому говорят, что ЧД — это область, выколотая из нашей Вселенной.

                        Вот на счет области выколотой из нашей вселенной я соглашусь, как и то, что за ГС невозможно провести никакую линию, так как, что-бы это сделать любая частица должна достичь бесконечной энергий в «собственной» как вы выразились СО. Но эта частица ни фига не достигнет бесконечной энергии потому, что согласно той же ОТО любая энергия это гравитационная масса, радиус которой равен Rs=2GE/c^4. И прикиньте в определенный момент(который так или иначе произойдет до достижения ГС), гравитационный радиус координатной энергии частицы/фотона неизбежно достигнет ее Комптоновской длины волны и тем самым превратит в черную дыру все, что падает на черную дыру. Что в таком случае произойдет с частицей и ее энергией можно только гадать, может ее изрыгнет обратно, а может будет создана новая вселенная). Но это точно будет не «незаметное и мирное пересечения ГС» с чего вообще и начинался этот спор)


                1. vanxant
                  06.10.2018 23:36

                  Так центр масс может быть весьма далеко от горизонта событий. Типичная сверхмассивная ЧД из центра какой-нибудь галактики имеет размер Солнечной системы по Нептун или типа того.


  1. Jogger
    05.10.2018 19:18

    Не знаю, в переводе дело или в исходной статье, но из объяснений непонятно ничего.
    И да, заголовок неверный — в школе не изучают квантовую механику вообще, поэтому это «10 физических фактов, которые вы НЕ должны были узнать в школе, и вероятнее всего не узнали».


    1. encyclopedist
      05.10.2018 19:22
      +1

      В российской школьной программе по физике для 11 класса есть разделы "Квантовая физика", "Корпускулярно-волновой дуализм", "Атомная физика", и "Физика атомного ядра".


      Но как показывают комментарии, многие эти разделы "проходили мимо".


      1. Jogger
        05.10.2018 23:07

        Видимо со времён моего обучения что-то поменялось. Даже интересно, действительно ли описанные выше факты есть в этом учебнике.


  1. Comod
    05.10.2018 19:33

    Но мы знаем, что в нашей Вселенной пространство расширяется, и это расширение нарушает закон сохранения энергии.

    Лютый бред, закону сохранения энергии во первых соответствует однородность времени
    Во вторых:
    Однако это нарушение настолько крохотное, что его не заметить ни в каком эксперименте, проводимом на Земле

    Мы это «не сохранение» наблюдаем каждыйбожийдень как гравитацию, кинетическая энергия упавшего камня определяется как энергия покоя камня умноженная на соотношение интервалов времени покоящихся часов, в начальной и конечной координатах камня минус энергия покоя — mc^2*(тау1/тау2-1)


    1. 4tlen
      05.10.2018 20:24

      Если бред, то вопрос: в расширяющейся вселенной два свободных протона удаляются друг от друга с ускорением. Они излучают?


      1. Comod
        05.10.2018 21:10

        В расширяющейся вселенной две свободных частицы на самом деле покоятся по отношению друг к другу, «не покоится» пространство между ними.


        1. vanxant
          05.10.2018 21:43
          -1

          Уравнениям Максвелла пофиг на детали. Есть ускоренное изменение расстояния между зарядами > есть переменное электрическое поле > есть вихревое магнитное поле вокруг переменного электрического > пошла электромагнитная волна.


          1. Comod
            05.10.2018 21:57

            Я вот не могу понять какую картину вы сейчас описали и к чему там расширение вселенной? Но если оно там к чему то есть, то почему оно должно быть ускоренным, если оно постоянное?


            1. 4tlen
              05.10.2018 22:01

              1. Comod
                05.10.2018 22:20

                Вы вообще в курсе, что это значит? Сейчас постоянная Хаббла в единицах СИ 2,17*10^-18 м/c на метр(с-1) что на 3 порядка меньше радиуса протона и в будущем она будет только уменьшаться в не зависимости от того расширяется вселенная с ускорением, или без.


    1. Gutt
      05.10.2018 23:43
      +1

      Мы излучили фотон. Он, двигаясь по расширяющемуся пространству, не содержащему другой энергии-материи, уменьшил свою частоту. Он, напомню, квант электромагнитного излучения, и нельзя сказать, что он «размазал» свою энергию по большей области пространства. Куда подевалась его энергия, зависящая от частоты?


      1. Victor_koly
        06.10.2018 00:13

        Все, закон с.э. сработал только локально.


      1. Comod
        06.10.2018 00:26

        Он, напомню, квант электромагнитного излучения, и нельзя сказать, что он «размазал» свою энергию по большей области пространства.

        Именно это и нужно сказать, учитывая то, что космологическое красное смещение имеет такое же отношение к Эффекту Доплера, на который вы типа намекаете, как морская свинка к морю.
        Куда подевалась его энергия, зависящая от частоты?

        Я сам не раз задавался себе этим вопросом, но тут только один кандидат для логичного ответа — темная материя — виртуальные частицы. И большая часть темной материи накопилась как раз в первые мгновения после БВ когда фотоны и не только «отдавали» энергию в пространство с огромной интенсивностью. Или спросите у физиков из чего состоит подавляющая часть массы протонов и нейтронов они вам явно начнут втирать про виртуальные частицы.


        1. Victor_koly
          06.10.2018 14:35
          +1

          Вне зависимости от факта наличия частиц (пар кварк-антикварк и глюонов) и их виртуальности энергия в принципе сохраняется. Действительно, в летящем на большой энергии протоне глюон может превратиться в пару t-anti-t или b-anti-b. Но для того, чтобы эти кварки стали реальными, они должны столкнуться с другой частицей и чтобы в системе ЦМ этих 2 протонов (ну или там нейтрон с одной стороны будет в ядре) энергии было достаточно.
          Или может будет столкновение W+-бозона (виртуально появляющегося в слабом взаимодействии) с b-кварком (из упомянутой выше пары).


  1. Winnie13
    05.10.2018 19:33

    Сложно сказать, можно ли упасть в чёрную дыру за конечное время.
    Ведь для стороннего наблюдателя не просто время падения объекта бесконечно, а и продолжительность формирования чёрной дыры тоже по идее бесконечно.
    В некотором смысле, их ещё нет этих чёрных дыр :) И никогда не будет :)


    1. Gutt
      05.10.2018 23:47
      +1

      Почему? После образования горизонта событий, действительно, ничего туда для внешнего наблюдателя окончательно упасть не может. Но что мешает формированию этого горизонта и его движению?


      1. 4tlen
        06.10.2018 00:01

        Там же бесконечное красное смещение и выход за видимый спектр, что можно назвать уходом под горизонт.


      1. Winnie13
        07.10.2018 15:50

        Ну я сразу сделаю оговорку, что я вообще ни разу не физик, а так, просто иногда размышляю над подобными граничными условиями для удовольствия.
        В моём понимании, чёрная дыра ж и формируется именно за счёт массы таких «объектов», которые в неё попадают. Допустим, до завершения образования горизонта события остался один квант вещества. Течение времени уже будет сильно замедлено в окрестности такой почти-чёрной дыры, и этому кванту как раз и потребуется ровно бесконечность, чтобы окончательно упасть в эту дыру и завершить формирование горизонта событий.
        Но если можете «по-пролетарски» объяснить, в чём тут провал в логике, то с удовольствием почитаю )


        1. DrSmile
          07.10.2018 17:58

          Квант будет лететь бесконечное время до старого горизонта, однако горизонт системы ЧД + квант будет немного отстоять от старого горизонта. В общем, время падения в ЧД объекта ненулевой массы конечно, даже с точки зрения бесконечно удаленного наблюдателя.


          1. taujavarob
            07.10.2018 20:56

            DrSmile

            В общем, время падения в ЧД объекта ненулевой массы конечно, даже с точки зрения бесконечно удаленного наблюдателя.
            Нет. Это оно (время падения) конечно только и только в гипотетических квантовых гравитационных теориях — коих наверняка много.

            В классической ОТО чёрная дыра, после своего образования, не может увеличиться ни на грамм, с точки зрения стороннего наблюдателя.
            Можно подсчитать, сколько времени займёт падение в чёрную дыру по часам падающего. Результат получается конечным. В чёрную дыру упасть можно. Просто ваш друг снаружи этого никогда не увидит.

            Тогда - допустим, вы (или некая цивилизация) появились в начале (более менее — лет так 13 миллиардов назад) Большого Взрыва и стали наблюдать Вселенную (ваши окрестности). — За это время вы увидите множество событий, в том числе и образование чёрных дыр по соседству или далеко (если у вас есть телескоп), но никогда не сможете увидеть как хоть один грамм вещества поглощается чёрной дырой.
            Вы можете видеть джеты излучаемые веществом при его поглощении чёрной дырой, вы можете видеть как чёрная дыра разрывает галактики и их материя устремляется к чёрной дыре и прочее и прочее — но вы никогда не увидите как хоть 1 грамм вещества будет поглощён чёрной дырой. Никогда! — Так утверждает классическая ОТО.

            P.S. Классическая ОТО — удивительная теория! — В ней не только нет внятного и физически имеющего смысл определения энергии, импульса, момента импульса (и конечно же законов их сохранения, даже есть проблемы с определением понятия массы!), — но теоретически даже гравитационные волны (за открытие которых недавно дали Нобелевку) не могут переносить энергию! Дело в ОТО обстоит так, что гравитационные волны передают «команду», то есть «информацию» о переносе энергии из одной точки в другую, но ни коем образом не саму энергию.


        1. 4tlen
          07.10.2018 20:22

          в чём тут провал в логике
          В том что время не замедляется для этого кванта. Эффект замедления видит только удаленный наблюдатель.


          1. Winnie13
            07.10.2018 21:35
            +1

            Так я и говорю только для внешнего наблюдателя.
            Квант провалился, по пути весело пронаблюдал в убыстренной съёмке историю всей Вселенной, и выключил свет :)
            А для стороннего наблюдателя он никогда (никогда за время Вселенной) не провалился, и никогда окончательно не сформировал чёрную дыру.


            1. 4tlen
              08.10.2018 11:23

              Если бы так, то этот квант, например фотон, вечно бы излучал для наблюдателя. Я выше написал что будет бесконечное красное смещение в радио диапазон и он перестанет быть видимым.

              image


              1. Winnie13
                08.10.2018 12:39

                ОК, немного путаница вышла. Под «квантом материи» я имел в виду минимальную массу материи, которая добавившись к почти-чёрной дыре превратила бы её в чёрную дыру. И которая будет добавляться вечно. Это было не про фотон, хотя мне сложно судить, фотон может тоже подходит для такого рассуждения (но вроде бы нет).
                И в любом случае — не вижу противоречия с красным смещением. Ну уходит оно бесконечно «вправо», и начиная с какого-то уровня перестают быть различимым для всех наших средств измерения. Ну и что? Всё равно излучение есть и будет длиться до скончания веков?


                1. BigBeaver
                  08.10.2018 12:59

                  Конечно подходят.


  1. SergeyMax
    05.10.2018 21:33
    +1

    Тяжёлые частицы распадаются не до состояния с минимальной энергией, а до состояния с максимальной энтропией

    Причина, по которой тяжёлые частицы распадаются, когда могут, состоит в том, что они это могут. Если у нас есть одна тяжёлая частица (допустим, мюон), она может распасться на электрон, мюонное нейтрино и электронное антинейтрино. Возможен и противоположный процесс, но для него требуется, чтобы в одном месте собрались три продукта распада. Следовательно, вероятность его мала.

    На вид очень интересное объяснение. Жаль, что оно не объясняет, почему атом кислорода самопроизвольно не распадается в состояние с максимальной энтропией, ведь для того, чтобы создать такой атом, нужно собрать в одном месте 48 кварков и 8 электронов!


    1. LynXzp
      06.10.2018 06:59

      Дефект массы -> потенциальная яма -> устойчивое равновесие. До ближайшего другого устойчивого равновесия очень далеко. Уран распадается потому что может (потому что ближайшее расстояние рядом, равновесие неустойчивое), дайте ему только полежать некоторое время или подтолкните нейтроном.

      Кислород самопроизвольно распасться не может еще потому что нет у более элементарных частиц состояния с меньшей энергией.
      image


      1. SergeyMax
        06.10.2018 07:36

        еще потому что нет у более элементарных частиц состояния с меньшей энергией.
        А автор статьи говорит, что энергия ни при чём, а всё дело в энтропии. Давайте вы с ним поспорите, а не со мной)


        1. DrSmile
          06.10.2018 19:01

          Просто энтропия зависит от энергии. Если при слиянии частиц выделяется энергия, то эта энергия (в конечном счете) переходит в кинетическую энергию результата. Больше кинетическая энергия — больше скорость — больше доступных микросостояний — больше энтропия.

          В общем, если уменьшение энтропии вследствие уменьшения числа частиц компенсируется увеличением энтропии вследствие возросшей скорости движения, то идет синтез, а не распад.


          1. BigBeaver
            06.10.2018 19:28

            То же самое в примере с маслом в воде, кстати.


  1. totally_nameless
    05.10.2018 22:47

    Энергия сохраняется. Поэтому идея о том, что любая система пытается минимизировать энергию, не имеет смысла.

    Ну все отменили фундамент физики, расходимся…


    1. Victor_koly
      06.10.2018 00:20
      +1

      любая система пытается минимизировать энергию

      Это просто единственный путь, каким может пойти система. Минимизировать свою энергию, выделив её куда-то наружу (или переведя в другую форму).
      Указанные выше 48 кварков не могут перейти в состояние с меньшей энергией (и 8 электронов им не помогут, это у калий-40 есть свыше 10% шансов на орбитальный захват, а тут по энергии не получается).


      1. totally_nameless
        06.10.2018 00:53

        Как можно изменить (минимизировать) энергию, просто переведя ее в другую форму?


        1. Victor_koly
          06.10.2018 13:56

          Скажем система испускает 1 или 2 фотона и переходит в форму, в которой её энергия (сумма кинетической и потенциальной) меньше, чем была 0.2 нс назад.


  1. Ilya81
    07.10.2018 12:56

    Причина, по которой тяжёлые частицы распадаются, когда могут, состоит в том, что они это могут. Если у нас есть одна тяжёлая частица (допустим, мюон), она может распасться на электрон, мюонное нейтрино и электронное антинейтрино. Возможен и противоположный процесс, но для него требуется, чтобы в одном месте собрались три продукта распада. Следовательно, вероятность его мала.

    Вот только интересно — протонам в приципе есть на что распадаться, но они не распадаются. В чём их уникальность?


    1. 4tlen
      07.10.2018 13:49

      В конфайнменте кварков.


    1. Victor_koly
      07.10.2018 14:55

      В принципе, протону нет на что распадаться (по закону с-я энергии), если мы считаем за аксиому сохранение барионного заряда. А инфу о том, что он не сохраняется, я у специалистов не спрашивал. То есть инфа есть, но я про неё не спрашивал.


  1. achekalin
    07.10.2018 17:22

    С названием непорядок: не в школе, а в вузе.


  1. Comod
    07.10.2018 18:34

    Всем смотреть
    У кого не очень с английским(как у меня типа) включайте субтитры и перевести на русский.

    спойлер
    2L/yv